Главная Юзердоски Каталог Трекер NSFW Настройки

Наука

Ответить в тред Ответить в тред
<<
Назад | Вниз | Каталог | Обновить | Автообновление | 549 62 161
Тред тупых вопросов Аноним 18/11/22 Птн 07:03:42 567887 1
16680330221090.jpg 261Кб, 811x608
811x608
Аноним 18/11/22 Птн 07:06:42 567888 2
Допустим потребление устройства 20 ампер при 240 вольтах
Но при 20 амперах сеть просядет до 220
Устройство со стабилизацией выходных параметров, начнёт тянуть 21.8 ампера
Сеть ещё просядет т.к. это уже не 20 ампер...
Как это сразу посчитать а не методом последовательных приближений?
Аноним 18/11/22 Птн 11:16:44 567898 3
image.png 39Кб, 642x644
642x644
Аноним 18/11/22 Птн 11:20:33 567899 4
Аноним 18/11/22 Птн 17:46:54 567912 5
>>567899
Чёт не то мне кажется.
Сначала упало 20
Это 10%
Затем упадёт 10% от 1%
Затем 10% от 0.1%
Затем 10% от 0.01%...
Ниже 217 оно не опустится
Аноним 18/11/22 Птн 18:30:53 567913 6
>>567912
Зачем тебе кажется - не кажется, есть закон Ома. Неоспоримая хуйня, работает как часы 300 лет.
>Затем упадёт 10% от 1%
Затем 10% от 0.1%
Затем 10% от 0.01%..
Это ты сам придумал?
Аноним 18/11/22 Птн 20:20:26 567915 7
KJpApCoyjaj2RY7[...].jpg 14Кб, 200x200
200x200
У меня сложнейший вопрос.
Представим что кто либо предложил дать мне по рофлу 0,998м3 кала в котором содержиться распиляный порошок 20 кг чистейшого золота.
Как можно было бы отделить порошок
Аноним 18/11/22 Птн 21:34:36 567918 8
Если разрыв между персонажами компьютерной игры и программистом настолько же велик, как между нами и богом, возможно ли его преодолеть в принципе и как это сделать?
Аноним 18/11/22 Птн 22:26:56 567922 9
>>567915
Тупо исправить и сжечь говно. Высушенное говно вполне хорошо горит.
Аноним 18/11/22 Птн 22:27:48 567923 10
>>567918
Сейчас нейросети уже умнее вебмакак.
Аноним 19/11/22 Суб 03:24:21 567927 11
>>567918
Захвати контроль над чипом и прикажи ему делать наномашины в реальном мире, собери из них себе тело и перенеси сознание.
Аноним 19/11/22 Суб 06:16:38 567934 12
>>567887 (OP)
Я слышал такие изыскания релятивистов, что мол скорость света может быть анизатропной и хуй проверишь. А разве это не должно сказываться на характере остывании тел? Ну скажем возьмем полированный шар из вольфрама, накалим его равномерно, поставим в космосе, и будем следить за скоростью остывания. Скорость света явно входит в закон Макса Планка, и анизатропия скорости света должна по идее должна привести к асимметричной температурной карте, не?
Аноним 19/11/22 Суб 06:53:11 567937 13
>>567934
Типа может меняться как угодно?
Так тут речь о том что она константа на которой всё во вселенной завязано, и "время", так что не заметишь
Аноним 19/11/22 Суб 07:13:58 567938 14
>>567937
Нет, время не завязано, это вообще не имеет никакого смысла.
Речь именно идет о том, что в одну сторону свет может лететь быстрее, в другую медленнее, измерить по лазерному лучу можно только среднее туда-обратно и среднее всегда равно с. При этом если туда с/2, а обратно бесконечно, то мы об этом не узнаем
Аноним 19/11/22 Суб 07:37:44 567939 15
>>567934
В модели чернотельного излучения считаются стоячие волны, зажатые между бесконечно удаленной точкой и точкой поверхности шара. Там волна должна как бы пройти от точки шара, отразиться от бесконечности и прийти обратно. Так что опять возникает херотень с туда сюда.
Аноним 19/11/22 Суб 09:07:47 567941 16
>>567939
Стоячая волна получается, при определенных граничных условиях, я вот сейчас напряженно вспоминаю волновое уравнение, скорость входит как параметр в диффуре, она разве на длину волны не вляет? То если в ту скорость одна, в другую другая, все равно волна стоячая?
Аноним 19/11/22 Суб 09:17:15 567943 17
>>567939
Я правильно понимаю, что предполагается анизотропия групповой скорости света в вакууме, а фазовая изотропна?
Аноним 19/11/22 Суб 11:07:07 567945 18
>>567941
Влияет, но анизотропия скорости света это больше про геометрию пространства-времени. Если перейти к волновому уравнению, то переменная скорость света будет как динамический параметр от координатор или ориентации базиса. В теории поля таки выделяют ориентацию базиса, чтоб не исключить инверсию времени.
Стоячая волна это особенная сумма двух решений, они могут быть даже в нелинейных средах.
>>567943
Фазовая тоже должна быть анизотропной, иначе дисперсия.
Аноним 19/11/22 Суб 11:21:15 567947 19
>>567945
>про геометрию пространства-времени
Я вот в ОТО вообще ниче не понимаю. Пространство абсолютно плоское в постановке гипотезы об анизотропии скорости, если длина волны туда выходит одна, а отраженная другая, то как вообще получится стоячая волна? языком попроще плз объясни.
Аноним 19/11/22 Суб 11:55:48 567950 20
>>567947
Геометрия это не только кривизна, помимо расхождение углов(строгое определение кривизны), есть расхождение расстояний, подобия и целый кучи параметров. Разность скорости по направлению это в первую очередь разные длины между двумя событиями.
Стоячая волна это особая сумма двух волн. Одна из них них не обязательно должна быть отраженной, могут быть два источника.
Аноним 19/11/22 Суб 12:10:27 567951 21
image.png 24Кб, 960x320
960x320
>>567950
Не, нихуя не понимаю. Тупой я. Давай проще.
Вот я под волну поставил линейку и по линейке смотрю. Слева у меня источник, справа зеркало. Чтобы я по линейке наблюдал стоячую волну, мне надо чтобы синяя двигалась с той же скоростью, что и красная. Если скорость будет другая, то узлы поплывут, либо другая частота должна быть.
А релятивисты мне говорят, что красная может относительно линейки двигаться с другой скоростью.
Аноним 19/11/22 Суб 12:11:05 567952 22
Mxj0.gif 571Кб, 960x320
960x320
Аноним 20/11/22 Вск 17:37:03 567980 23
Аноним 20/11/22 Вск 20:11:55 567989 24
Почему в самолёте закладывает уши, если он герметичный и в нем установлена система поддержки давления (ака во всех современных крупных самолётах)? Аналогично почему подводники могут страдать от компрессионной болезни (или как она там называется), если вода давит на жесткий корпус, который воспринимает все давление, а внутри опять же поддерживается нормальное давление?
Аноним 20/11/22 Вск 20:27:24 567990 25
Я тут мучаясь от бессонницы прошлой ночью пролистал всю доску до конца, читая интересные треды. Мне кажется, или у вас тут в какое-то время срал один конкретный шиз однотипными тредами и постами? Иногда даже в нормальном по слогу отдельных постов было видно.
Аноним 20/11/22 Вск 20:33:39 567991 26
>>567989
>если он герметичный
Еще один в гугл не умеет, а всё туда же лезет.
Аноним 21/11/22 Пнд 02:31:06 568006 27
Как называется дисциплина, которая изучает жизнь других народов? Причем не только страрую хуйню, вроде традиций, религий и народной одежды, но и современный быт, политическое и социальное устройство, современную культуру и т.д.
Аноним 21/11/22 Пнд 02:34:23 568007 28
>>568006
>Причем не только страрую хуйню, вроде традиций, религий и народной одежды, как в этнографии; но и современный быт, политическое и социальное устройство, современную культуру и т.д.
Обосрался
Аноним 21/11/22 Пнд 03:20:11 568010 29
>>567913
>Зачем тебе кажется
Я считал, по закону Ома, но походу не то кое-где подставил.
Аноним 21/11/22 Пнд 03:23:10 568011 30
>>567990
Их несколько было, один тупой шиз, а другой тупая шалава которая считала что "чувства и ощущения" это дохуя наука
Аноним 21/11/22 Пнд 12:00:21 568023 31
С какой скоростью передётся движение в твёвдом теле? У нас есть прямоугольник, толкнули с одной стороны, когда противоположная сторона начнёт двигаться?
Аноним 21/11/22 Пнд 12:08:46 568024 32
>>568023
Естественно мгновенно, это ж твёрдое тело
Аноним 21/11/22 Пнд 12:21:47 568029 33
>>568023
Со скоростью звука для данного материла.
Если тело сложной формы и неоднородное, то чуть меньше скорости звука, то примерно такого же порядка.
Аноним 21/11/22 Пнд 13:39:03 568033 34
>>568023
Не слушай додиков, алсо можешь толкнуть детскую пружинку и проверить кукарек про скорость звука и порядки.

Скорость будет зависить от упругости материала (ака модуля Юнга) и от геометрии.
Аноним 21/11/22 Пнд 14:12:18 568038 35
>>568023
>С какой скоростью передётся движение в твёвдом теле?
Это на самом деле не такой простой вопрос как кажется, потому что зависит от условий в котором это тело находится.
Аноним 21/11/22 Пнд 14:23:14 568040 36
image.png 256Кб, 382x608
382x608
>>567915
Говно не тонет. Золото тонет.
Ебашишь золотоносное говно в тазик с водой умеренными порциями, перемешиваешь, даёшь хорошо отстояться, черпаешь со дна.
Аноним 21/11/22 Пнд 14:25:55 568041 37
>>567918
А не боишься, что твои персонажи настучат тебе по ебалу, когда преодолеют разрыв?
Представь ситуацию, что ты в реальной жизни обращаешься с людьми как с игровыми персонажами. В принципе это очень близкое моделирование ситуации. Как скоро ты получишь по ебалу?
Аноним 21/11/22 Пнд 14:51:05 568043 38
image.png 658Кб, 1500x682
1500x682
>>567887 (OP)
Какие доказательства, что ихтиозавры это рептилии?
Аргументы против рептильности:
Череп амфибиный. Шейный отдел позвоночника короткий как у прогрессивных, но всё же амфибий. Яичной скорлупы нет. Конечности не приспособлены для передвижения по суше. На суше не жили ни как. Сухопутные рептильные предки не найдены.
Аноним 21/11/22 Пнд 14:56:05 568044 39
image.png 232Кб, 580x435
580x435
>>567887 (OP)
Какие ваши доказательства, что в бронзовом веке Афины были цивилизоваными, городом и вообще греческими?
Почему не найдено ни одной надписи линейным письмом в Афинах?
Аргументы против: все культурные артефакты и руины в Афинах относятся к послебронзовой эпохе.
Аноним 21/11/22 Пнд 20:21:11 568060 40
Аноним 22/11/22 Втр 03:06:12 568078 41
>>568060
Он не герметичный
И давление там системы обеспечивают только как на довольно высокой горе, 3-5км, когда летает на 10-12км.
Аноним 22/11/22 Втр 03:09:16 568079 42
>>568078
> Он не герметичный
Подводная лодка тоже не герметична, клоун?
Аноним 22/11/22 Втр 03:29:21 568080 43
>>568079
Ты чего порвалась дурочка тупая?
Аноним 22/11/22 Втр 03:31:20 568081 44
Аноним 22/11/22 Втр 06:48:37 568084 45
>>567989
Чтобы снизить вероятность разрыва сраки повреждения корпуса от давления (а то вдруг в турбулентности затрясёт), в салоне поддерживается давление воздуха такое, как на вершине горы 4 км. Этого достаточно чтобы заложило уши. Когда сам самолёт летит на высоте 9 км.
Да блин, чтобы уши заложило, достаточно даже быстро спуститься с высоты 300 метров. Кто катался в горах на лыжах или сноуборде, тот в курсе.
Аноним 22/11/22 Втр 07:55:03 568086 46
>>568084
Основная причина в том что пассажирам нужно чем-то дышать, а так сильно сжимать воздух слишком заёбно, поэтому, соответственно надышанный воздух выкидывают за борт, и берут новый из-за борта, а там, внезапно, сильно разряженный, и нагнетают турбиной(они очень херовы в сильном сжатии, чтобы 1 атмосферу сделать уже нужен поршневой или виновой компрессор, а у них слишком маленькая производительность) или из какой-то ступеней движка берут
Аноним 22/11/22 Втр 08:44:07 568092 47
>>568086
>пассажирам нужно чем-то дышать
Спасибо, кэп.
Аноним 22/11/22 Втр 08:48:47 568093 48
>>568086
Можно хоть 10 атмосфер накачать. Турбина двигателя справится. Но зачем. Смысл как раз в том, что разница между внутренним давлением и внешним была минимально возможной, при которой человеки не синеют от нехватки кислорода. Чтобы самолёт не порвался как воздушный шарик при попадании в турбулентность.
Самолёты же, особенно пассажирские - очень хлипкие конструкции. У них всеми возможными способами снижают массу и вместе с массой снижается прочность конструкции до минимума, необходимого чтобы просто долететь из точки А в точку Б и не развалиться по пути.
Аноним 22/11/22 Втр 08:49:09 568094 49
>>568092
Ну даунчик сверху(не прям этот, а ещё выше) об этом не догадался, рьяно думал что самолёт герметичный. Да и этот который выше о другом писал.
Аноним 22/11/22 Втр 08:51:45 568095 50
>>568094
Герметичность понятие относительное. Самолёт не такой герметичный, как например балон с водородом (из которого всё равно просачивается водород по чуть, хоть как его ни запаивай). Но всё же достаточно герметичен чтобы держать в себе давление воздуха выше чем снаружи при условии постоянной подкачки.
Аноним 22/11/22 Втр 09:04:10 568100 51
>>568095
Очень тупые отпёздки.
Труба сквозная без дна тоже герметична, т.к. вкачивая в неё достаточно можно создать пиздецовое давление.
Самолёт нихера не герметичен, воздухообмен огромный относительно его объёма.
Аноним 22/11/22 Втр 09:38:44 568103 52
>>568100
Хорошо, как скажешь. Это всего лишь разные точки зрения на одно и то же. Обе верны.
Аноним 22/11/22 Втр 12:53:27 568109 53
СЛУШАЕШЬ РУССКОЯЗЫНОГО НАУЧПОПА
@
КОРЕННЫЕ АМЕРИКАНЦЫ
@
СМОТРИШЬ ИСТОЧНИКИ ФИНАНСИРОВАНИЯ ЕГО КОНТОРЫ
@
СОРОС


Нахуй так жить?
Аноним 22/11/22 Втр 13:01:09 568110 54
>>568109
Финансируй какой нужно научпоп сам.
Аноним 22/11/22 Втр 13:02:21 568111 55
>>568109
Сколько ты сегодня бомжу сосал в мусорке представляя что сосёшь пыне?
Аноним 22/11/22 Втр 13:34:36 568112 56
>>568109
СЛУЧАЕШЬ РУССКИЙ НАУПОП
@
ГМО КОМАРЫ РУСОФОБЫ С ПЕЧАТЬЮ СОТОНЫ
@
ОБЛУЧАТЕЛЬ ГЕЙ ИЗЛУЧЕНИЯ 5 ДЖИ
Аноним 22/11/22 Втр 13:53:33 568113 57
Прикиньте, в ютубе нет ни одного видео о том почему аллюминий не магнитится
Аноним 22/11/22 Втр 15:33:11 568115 58
>>568113
Алюминий же парамагнетик, так что он вполне магнититься.
Парамагнетизм и диамагнетизм проходят в вузовских курсах общей физики, а их лекции точно должны быть на ютубе.
Аноним 22/11/22 Втр 15:36:37 568116 59
>>568115
>парамагнетик
Типа очень слабо? Так что не заметно почти
Или ты про то что он взаимодействийствует с магнитом потому что он проводник и в нём токи наводятся
Аноним 22/11/22 Втр 15:36:52 568117 60
Анон, а есть ли годное попсовое видео по синтезу аминокислот в организме?
Типа такого. А то про белок куча простых видео, а про аминокислоты сразу какая-то окололекционная нуднятина.
https://www.youtube.com/watch?v=agLNVS3BM3w
Аноним 22/11/22 Втр 15:47:04 568118 61
Есть ли в природе пример идеально равномерного движения? С учётом переменной нагрузки этому движению, т.к. идеально равномерное движение несмотря на меняющуюся нагрузку.
Без всякого треша типа мысленного эксперимента с инерцией тела бесконечной массы

Вот как например в обратном примере с трением покоя, пока сила, как бы она не менялась и не колебалась, меньше некотрой величины, тела не сдивнется.
Аноним 22/11/22 Втр 15:55:26 568119 62
>>568116
Нет, именно парамагнетизм - втягивание в неоднородном стационарном поле.
Чтоб стало заметно нужны достаточно сильные поля больше 1 Тл.
Аноним 22/11/22 Втр 16:35:24 568120 63
>>568119
Т.е. нету полностью немагнитных металлов?

И ещё, вот допустим у аллюминия низкое сопротивленик, поэтому магнит в алюминиевой трубе падает медленно
А железо магнитится сильнее, а сопротивление больше.
В стальной трубе магнит упадёт быстро.
Есть ли какой-то металл, который магнитится как железо, а с сопротивлением как у алюминия? Или магнитится сильнее а сопротивление меньше. Или магнитится слабее а сопротивление больше.

Т.е. чёт кажется что это прям жёстко завязано.
Аноним 22/11/22 Втр 20:38:10 568127 64
>>568120
Ну любое вещество магнитно активно, вопрос только в степени и характере. И эта активность не связана с проводимостью вещество, образуя самостоятельное явление.
Железо магнититься не за счет того, что он металл, а из-за его особенности кристаллической решетки, где изолированные электроны ориентируют свои спины вместе.
>Есть ли какой-то металл, который магнитится как железо, а с сопротивлением как у алюминия?
Как чистый элемент вряд ли, разве что в не привычных условиях как давление и температура.
Аноним 22/11/22 Втр 22:51:26 568135 65
>>568112
Тащемта речь про Соколова шла. Пиздец как я расстроен.
Аноним 24/11/22 Чтв 02:54:04 568205 66
Возможно ли создание несложно устроенного искусственного лица, умеющего в простую мимику, пародирующее устройство лица человека в общих чертах, но не стремящееся его скопировать?
Типа как аниматронное лицо робота из фильмов 60х, но не такое убогое. Не силиконовое как пытаются делать сейчас.
Аноним 24/11/22 Чтв 04:56:00 568206 67
>>568205
>Не силиконовое
А какое тогда? Чем тебе силикон не угодил?
Аноним 24/11/22 Чтв 07:23:29 568223 68
234845774l0PtoS[...].jpg 48Кб, 596x387
596x387
Аноним 24/11/22 Чтв 07:32:12 568224 69
>>568223
Но ведь силикон не горит.
Аноним 24/11/22 Чтв 07:45:47 568225 70
>>568224
Формально горит, он выделяет больше при соединении с кислородом чем требуется для начала соединения.

>>568223
Походу нету мягких/гибких вешеств у которых отрицательная энергия связи с кислородом.
Аноним 24/11/22 Чтв 08:23:18 568230 71
>>568225
Формально всё горит кроме, кислорода, фтора и инертных газов. Озвучте уж тогда технические условия. Предельные температуры хотя бы.
Аноним 24/11/22 Чтв 09:54:31 568237 72
>>568230
Золото же не горит вроде, да и много чего ещё.
Его оксид распадается наоборот с выделением энергии.
Аноним 24/11/22 Чтв 09:59:12 568238 73
Слушьте, вот в нашей вселенной была в начале аннигиляция в соотношении 100 частиц антиматерии к 101 частице материи.
Получается не существует такого соотношения, чтобы было в точности наоборот?
Типа антивселенная есть или невозможна?
Это получается тут было 50.5 частиц матери и 50 частиц антиматерии, а в антивселенной было 50.5 частиц антиматерии и 50 частиц материи
В сумме получается 100.5 антиматерии и 100.5 материи
А, ну в принципе получается...
Аноним 24/11/22 Чтв 13:33:48 568245 74
>>568238
Причина возникновение барионной асимметрии до сих пор не ясна, но ясно одно точно, то что возникла после эпохи инфляции. Это не закрывает возможность для антивселенных, но сильно снижает ее возможность. По наиболее популярной модели материи стало чуток больше из-за асимметрии по слабому взаимодействию, так антивселенные невозможны.
Аноним 24/11/22 Чтв 14:24:18 568250 75
>>568245
>Это не закрывает возможность для антивселенных, но сильно снижает ее возможность.
Сфигали? Наоборот вроде делает её существование более логичным.

Вот только с чего вообще взяли что ассиметрия была...
Аноним 24/11/22 Чтв 15:02:35 568252 76
Что будет, если ионизировать водород?
В моём представлении, он лишиться единственного электрона, и... ?
Исчезнет, распадётся на составляющие, мутирует в гидралиска?
Аноним 24/11/22 Чтв 15:49:03 568254 77
>>568252
Протон стабилен, имеет заряд +1

А вот что с ним будет потом если его просто выкинуть в окружающую среду хз
Сильнее ли он пытается забрать себе электрон у каких-то других атомов или слабее
А может вообще пытается встроеться в какое-то химическое вещество не имея электрона.. Чтобы не забрать себе электрон у атома какого-то, а образовать целиковое неполное одним электроном химическое соединение

Также непонятно что с ним физически а не хичически будет, вот водород вверх полетит в атмосфере, а протон один, он как, тяжёлый, в землю упадёт, или лёгкий как водород, хз
Аноним 24/11/22 Чтв 15:57:07 568255 78
>>568252
Протон с нейтроном таки притягиваются, нужно брать версию водорода без нейтрона имя забыл.
Аноним 24/11/22 Чтв 16:24:24 568256 79
>>568230
>>568225
>>568237
>>568224
Стали будет достаточно. Вопрос очевидно в том, реально ли сделать лицо классическому роботу полностью из металла, чтобы лысой обезьяне было комфортно смотреть на него и чтобы в то же время мимика такого лица могла выражать эмоции.
Аноним 24/11/22 Чтв 17:31:34 568257 80
>>568254
>>568252

Посоны, а в космосе водород летает в какой форме? Он там молекулярный или ионизированный? Или в разных областях по разному?
Аноним 24/11/22 Чтв 18:10:24 568259 81
>>568257
Естественно в разных областях по разному.
Если брать среднем по Вселенной, то больше половина вещества это нейтральный атомарный водород и гелий.
Если брать в среднем по галактике, то тут уже зависеть от галактики. В галактиках с активным звездообразованием водород уже преимущественно в молекулярной форме.
В межзвездной среде в галактике же преимущественно уже ионизированный водород.
В окрестностях Солнечной системы так вообще водород считай как сверх горячая разряженная плазма.
Аноним 24/11/22 Чтв 18:27:28 568261 82
>>568254
Он просто цепляется к любой хуйне с электронами и сидит на нём. Может конечно отобрать себе электроны, но и сидеть на электронном облаке ему тоже норм. Так кислоты же работают, просто протон перекидывают друг-другу
Аноним 24/11/22 Чтв 18:39:29 568262 83
>>568259
>В межзвездной среде в галактике же преимущественно уже ионизированный водород.
Второе понятно почему без электронов, а в этом случае где электроны?
>В окрестностях Солнечной системы так вообще водород считай как сверх горячая разряженная плазма.
И насколько горячая? Синтез идёт или нет
Аноним 24/11/22 Чтв 18:52:34 568265 84
изображение.png 35Кб, 607x600
607x600
Продублирую вопрос тут, мб кто ответит в двух словах. Короче, задача: запилить максимально простой алгоритм рассчитывающий физику твердого тела (куба, для простоты) при столкновении с рельефом местности (набор треугольников по квадратный сетке с разными высотами). Цель сделать что-то визуально похожее на правду.

Думаю, попробовать сделать так:

При пересечении одной из точек объекта плоскости препятствия задать в этой точке вектор силы, который будет направлен перпендикулярно плоскости препятствия. Потом этот вектор разложить на тангенциальную и нормальные составляющие относительно вектора между точкой столкновения и центром масс объекта, и применить тангенциальную к импульсу, а нормальную к моменту импульса (все рассчеты будут производиться не с точками, а центром масс - перемещение и вращение объекта будут являться следствием сил, действующих на центр масс).

Оцените херовость затеи по десятибальной шкале: насколько вероятно, что оно заработает так, как мне хочется?
Аноним 24/11/22 Чтв 19:06:25 568266 85
>>568262
Рядышком в паре километров летает где-нибудь, лол.
Водород может захватить электрон, став отрицательно заряженным. Но чаще всего электроны собирают на себе какие-нибудь пылинки чисто за счет электроемкости.

Солнечная корона за лимон градусов и это в спокойном режиме. Теоретически синтез может идти, но длина свободного пробега у протона в ней измеряются астрономическими величинами. В добавок поток частиц порождает мощные магнитные поля, которые препятствуют столкновениям между собой.
Аноним 24/11/22 Чтв 19:12:58 568267 86
>>568265
Хуита.
Не изобретай велосипед, используй линейные итерации для коллизий. А там уже используй матричные операции для пересчета вращательного момента.
Аноним 24/11/22 Чтв 19:23:06 568269 87
>>568267
Можешь скинуть ссылку на 1-2 статьи по теме? Что-то то, что я нахожу, не слишком подходит под описание задачи. Ну и еще: у меня реально нет цели сейчас сильно задрачивать - цель, тупо сделать за минимальное время (включая изучение инфы) так, чтобы выглядело хотя бы отдаленно похоже. Если, вдруг, то, что я написал удовлетворяет этим условиям, то, пока что, нахуй другое (но инфу все равно, плс, скинь, если не сложно - когда-нибудь доберусь, но не сейчас)
Аноним 24/11/22 Чтв 19:34:30 568270 88
>>568269
Сходи на сайты игроделов и вкуривай алгоритмы расчет коллизий, все равно они сводятся к расчету динамики твердого тела.
Аноним 24/11/22 Чтв 19:42:02 568272 89
>>568270
>Сходи на сайты игроделов и вкуривай алгоритмы расчет коллизий
В любом другом случае так бы и сделал, но на то, чтобы "хуяк-хуяк и в продакшн" у меня рили пара дней-неделя. Так что все, что не на поверхности, временно отметается. Позже уже буду думать, как сделать нормально.
Аноним 25/11/22 Птн 03:05:15 568283 90
>>568256
Ну делают же красивые театральные и мультяшные куклыварешки с мимикой. Правда их делают из говна и палок. Но можн оиспользовать и металл. Главное красиво раскрасить.
Или у тебя требования чтобы был только лишь металл и никакой краски, штоб краска не горела?
Аноним 25/11/22 Птн 04:29:59 568284 91
>>568283
Можно и с краской. Просто стало интересно — реален ли ИРЛ металлический робот как из мультфильмов. Никогда ни медиа ни в артах не видел таких, поэтому мой мозг в тупике, не могу представить.
Аноним 25/11/22 Птн 16:34:28 568299 92
Допустим у дерева 15МДж теплота сгорания на кг
Допустим оно на 100% сгорело

Допустим я охладит продукты сгорания с 600 градусов цельсия до 100 градусов перед выкидыванием.
Допустим на улице -10 а в помещении +20

Это сколько я энергии из них при этом извлеку?
Как понимаю в справочниках указана теплота сгорания относительно абсолютного нуля, хотя если я буду пересчитывать относительно абсолютного нуля, то нахуй мне дрова, я получается с таким же успехом пересчитывая относительно абсолютного нуля смогу черпать бесконечно энергии из уличных -10...
Хотя не, охлаждать же могу только до 100 цельсия

Как бы логично чтобы в справочнике было относительно абсолютного нуля, но чёт я сомневаюсь что для каких-то дров заморчивались в справочнике пересчётом к абсолютному нулю или тем более эксперментом не пытались как-то условия к такому приблизить.
Аноним 25/11/22 Птн 16:39:21 568300 93
В принципе, теплоёмкость самого топлива относительно его количества и энергии сгорания в нём очень маленькая и почти ни на что не повлияет.
И если кислорода поступило ровно столько сколько нужно для сгорания тоже не повлияет почти.

А если точно известно их количество и температура, то начальная температура от того что они находились в окружающей среде которая теплее абсолютного нуля, допустим +20, даст ровно такой же плюс энергетический, насколько же не прлучится озладить продукты сгорания ниже огружающей среды.
Аноним 25/11/22 Птн 17:20:32 568301 94
Хм, с одной стороны, теплоёмкость топлива относительно энергии в нём на массу очень низкая...
С другой стороны, вся энергия может быть только в температуре продуктов сгорания
И допустим если температура и топлива и окислителя +20 цельсия начальная
А потом продукты сгорания с 600 до 100 охлаждаются
То 20 к 600 довольно значительно
Аноним 25/11/22 Птн 18:35:45 568306 95
>>568299
>Как понимаю в справочниках указана теплота сгорания относительно абсолютного нуля
Там для стандартных условий (давление 1 бар, температура 25 цельсия). От того, что твои продукты остынут до 100 а не до 25 теплота почти не изменится, посмотри на теплоемкость продуктов и поймешь.
Аноним 25/11/22 Птн 19:20:32 568309 96
>>568306
>посмотри на теплоемкость продуктов и поймешь.
Надо ж не на теплоёмкость смотреть а на разницу начальная/конечная температура относительно этих 25 цельсия
Так-то вся энергия выделившаяся в любом случае будет в продуктах
Или ты про то что там у них температура сильно выше будет после сгорания начальная?
Так-то у меня по факту всего 600 получилось по цельсию, конечно тут наверное подмес окислителя сильно с большим запасом и он на себя много забирает, да и в окислителе кроме самого окислителя много лишнмх примесей(азот) нагрев которых впустую идёт с 20 градусов, но всё равно, 600 как-то не много совсем, разница 20/100 существенная
Аноним 25/11/22 Птн 20:46:52 568314 97
>>568309
>Или ты про то что там у них температура сильно выше будет после сгорания начальная?
Это не важно, теплоту указывают для остывших продуктов. Если же у тебя продукты горячие (600 градусов), то просто часть теплоты сгорания уйдет с ними.
Аноним 26/11/22 Суб 08:21:21 568322 98
>>568314
Ну так причём тут теплоёмкость
Аноним 26/11/22 Суб 08:31:29 568323 99
Пусть есть функция скорости от времени. Если взять неопределенный интеграл от функции скорости, то мы получим функцию координат, чтобы в любой момент знать где находимся. Если взять неопределенный интеграл, то мы получим площадь под графиком скорости, где будет понятно, что значение площади от 0 до времени t есть то, как далеко уехала точка от 0 момента времени (при условии что график скорости все время положительный). Если предположить, что точка отсчета координат в момент времени 0 и точка в одном месте, то неопределенный интеграл в момент t даст то же самое, что и определенный от 0 до t? Просто в одном случае мы знаем, на сколько переместились, а во втором свою координату в моменте t и если перемещение было от 0 то они совпадут?
Аноним 26/11/22 Суб 09:08:47 568327 100
149957233614995[...].jpg 109Кб, 409x454
409x454
Где искать научные работы по темам вроде:
Склонность к социопатии у потомков продажных женщин.
Склонность к геноцидам у плодов близкородственного кровосмешения.
Склонность к шовинистическим настроениям у людей с высокой чивствительностью или большим уровнем норадреналина.
Нацизм как проявление комплекса этнической неполноценности у метисов.
Аноним 26/11/22 Суб 13:47:34 568334 101
Возвращаясь к сжиганию дров
Допустим я замерил равновесную температуру до теплообменника, 600 градусов
Но наверняка у меня идёт сильный подмес лишнего воздуха, а на его нагрев соответственно
В помещении всё так же +20, воздух соответственно тоже +20
Получая имея данные:вода на входе теплообменика, вода на выходе теплообменника, продукты сгорания на выходе, продукты сгорария до теплообменника, и воздуха +20
я могу рассчитать реальные потери даже не зная расход воздуха чтобы знать сколько его там лишнего
Т.к. на лишнее его количество, на нагрев его уже будет тратиться энергия, с +20 до уравновешившейся температуры +600 до теплообменника. И потом эта его оишняя масса, больше массы но нагретой до меньшей температуры, будет давать такое же количество энергии в теплообменник при меньшем падении температуры, с 600 до 80 например.(а так бы было, если бы лишнего воздуха не было, было бы с 900 до 80 например, а энергия отданная теплообменнику та же)
Аноним 26/11/22 Суб 17:58:46 568355 102
Как начать изучать матан с нуля? Качать школьные учебники?
Аноним 26/11/22 Суб 18:18:41 568357 103
>>568355
Сначала понять зачем...
А так есть специальная доска для этого.
Аноним 26/11/22 Суб 18:31:00 568359 104
>>568357
>Сначала понять зачем...
Причин много
>А так есть специальная доска для этого
Спс, не заметил сначала, сейчас нашел
Аноним 26/11/22 Суб 18:39:50 568361 105
Объясните, как возможно существование иррациональных чисел, если наука заявляет об исключительной рациональности нашего мира? Если математика точная наука, как она может вмещать в себя рациональное и неподдающееся конечному вычислению? О чём это говорит?
Аноним 26/11/22 Суб 18:53:34 568364 106
>>568361
>О чём это говорит?
О том что ты тупой фантазёр.
Наука ни о чём не говорит, у неё есть научный метод, на этом всё.
Математика кстати тоже построена на чём-то типа научного метода.
Аноним 26/11/22 Суб 19:00:02 568365 107
>>568364
Все учёные и вы здесь заявляете о не существовании бога.
Спрошу иначе. Откуда в точной науке берётся неточность, не являющаяся ошибкой? Как может научно существовать парадокс?
Аноним 26/11/22 Суб 19:14:12 568369 108
>>568365
>Откуда в точной науке берётся неточность, не являющаяся ошибкой?
Из фундаментальной природы реальности. Всё зависит от положения. Но где именно какой-то объект? Чуть ближе, или чуть дальше
Приходим к бесконечнодробности при определении положения.
А от положения зависят уже и взаимодействия объектов, а это уже вся физика.
Ок, решили ввести некую фундаментальную "сетку".
Но взаимодйствие зависит от расстояния, и находясь не в точности на одной линии сетки, а в её углах, при определённом соотношении будет получаться нецелочисленное деление бесконечнодробное.
>Как может научно существовать парадокс?
Что значит парадокс? Ты типа парадоксом называешь неточность?
Парадоксом обычно называют то, что у большинства на первый взгляд кажется "что так, а на самом деле вот так".
Т.е. парадокс это то у чего есть решение, но неочевидное на первый взгяд.

>Все учёные и вы здесь заявляете о не существовании бога.
Это не соответствует научному методу, можешь просто выписывать их из учёных.
Хотя возможно они просто засирают твоё понимание о боге которое не соответствует научному методу, или твои тупые попытки говорить о боге вне научного метода, тогда сам дурак.
Аноним 26/11/22 Суб 21:03:29 568377 109
Что будет если человек будет дышать как собака. Высунув язык и часто
Аноним 27/11/22 Вск 00:30:37 568396 110
Есть какие-то теории, где внятно объясняется как "внутри" работает квантовая запутанность?

Объяснение по КМ, что это магия, не катит.
Аноним 27/11/22 Вск 01:44:52 568399 111
>>568396
КМ объясняет отлично и без магии. Что тебе непонятно в объяснении КМ?
Аноним 27/11/22 Вск 12:05:58 568402 112
>>568369
>Из фундаментальной природы реальности. Чуть ближе, или чуть дальше
>Приходим к бесконечнодробности при определении положения.
То есть не существует точного описания реальности. Это потому, что математическая модель основана на нашем несовершенном восприятии? Или выражение в числовом диапазоне не даёт определённости?
>А от положения зависят уже и взаимодействия объектов, а это уже вся физик
Мы можем делить пространство до тех пор, пока на субатомном уровне не начнут работать законы квантовой физики и механика уже не будет давать результата, потеряет смысл сама по себе. То есть попытка точного вычисления невозможна и теряет смысл в определённом моменте.
>Что значит парадокс? Ты типа парадоксом называешь неточность?
Любой в принципе, как пример выше.
>Это не соответствует научному методу
Что соответствует?
Аноним 27/11/22 Вск 12:51:26 568403 113
>>568402
>не начнут работать законы квантовой физики и механика уже не будет давать результата, потеряет смысл сама по себе.
Ты не можешь знать существует квантовая механика и какие механизмы её работы, и нет никакой потери смысла, есть фундаментальная нерешаемая проблема бесконечной точности.
>Любой в принципе, как пример выше.
Это не парадокс.
>Что соответствует?
Проверяемое и доказуемое и имеющее доказательство соответсвующее критериям проверяемости и доказуемости.
Аноним 27/11/22 Вск 14:14:12 568405 114
Я правильно понимаю: гальванический элемент выделяет энергию за счёт создания анионов?
Аноним 27/11/22 Вск 15:25:42 568406 115
>>568361
> Объясните, как возможно существование иррациональных чисел, если наука заявляет об исключительной рациональности нашего мира?
По моему термины рациональный мир и рациональное число это несколько разные и непересекающиеся вещи.
Аноним 27/11/22 Вск 15:27:16 568407 116
>>568396
> Есть какие-то теории, где внятно объясняется как "внутри" работает квантовая запутанность?
Нету вроде бы.
> Объяснение по КМ, что это магия, не катит.
По км магии нет, есть строгая математическая зависимость, основная на вероятностях.
Аноним 27/11/22 Вск 15:31:25 568408 117
>>568405
>Я правильно понимаю: гальванический элемент выделяет энергию за счёт создания анионов?
Нет.
Гальвавнический элемент использует энергию окислительно-восстановительной реакции.
В ходе окислительно-восстановительной реакции передаются электроны от восстановителя к окислителю.
Если окислитель и восстановитель разделены в пространстве, их можно соединить проводником, по которому потечет ток.
Аноним 27/11/22 Вск 15:34:40 568409 118
>>568406
>непересекающиеся вещи.
Неправда. Исторически это связанные вещи.
Просто мир оказался сложнее, чем представлялось рационалистам 500-летней давности.
Сегодня, разумеется, "рациональное число" - это термин, а иррациональные числа вполне укладываются в разуме любого здорового индивида.
Аноним 27/11/22 Вск 15:57:19 568412 119
>>568409
> >непересекающиеся вещи.
> Неправда. Исторически это связанные вещи.
> Просто мир оказался сложнее, чем представлялось рационалистам 500-летней давности.
Предполагалось, что мир описывается рациональными числами, и поэтому его назвали рациональным миром?
Аноним 27/11/22 Вск 16:24:08 568414 120
>>568408
1)Гальванический элемент создает разницу потенциалов в цепи.
2)Разница потенциалов ведет к тому что электроны в цепи начинают двигаться. Но они движатся чуть чуть а не весь путь от - к +. Это движения по закону большого пальца создает момент электромагнитного поля направленый к цепи и цепь начинает работать. Разница потенциалов исчезает организованное движение электронов прекращается и момент исчезает. Цепь перестает работать.
Аноним 27/11/22 Вск 16:57:53 568416 121
concerned-cat-k[...].gif 11978Кб, 640x640
640x640
Изнашивается ли влагалище женщины после множества пенетраций мужскими половыми хуями и другими подручными предметами?
Конечно, любой любитель научпопа и просто человек с базовой медицинской грамотностью скажет, что это чепуха, и добавит, что влагалище расширяется после родов. Или с возрастом, за десятилетия.
Но я пот послушал и почитал гинекологов и сексопатологов, и вне популярных лекций они прямо этого не говорят. Т.е. даже прямой вопрос просто обходится, сразу перескакивают на роды, иногда говорится, что "никакой член так не изнашивает, как роды".
В связи с этим вопрос: наука таки доказала или не доказала?
Аноним 27/11/22 Вск 17:01:54 568417 122
>>568416
Ну смотри, мышцы как и рэле могут быть нормально разомкнутыми и нормально замкнутыми.
Для начала узнай есть ли там вообще мышцы и какой вообще их вклад если есть они там в узость влагалища.
Аноним 27/11/22 Вск 17:06:44 568418 123
>>568417
>Для начала узнай есть ли там вообще мышцы
Я читал, что есть. И не только мышцы, много всего. Ткани всякие. Такова научная теория. Я на картинках видел. И даже видео смотрел. Да что там, я даже находился вблизи женщины. Буквально на расстоянии вытянутой руки - и не раз, не знаю, поверишь ли мне.
Я же спрашиваю про прикладную науку.
Аноним 27/11/22 Вск 17:40:52 568419 124
>>568416
Анон мы споткнемся уже на том моменте какому из 26 полов принадлежит оное влагалище.
Потом мы столкнемся со скептецизмом по поводу способности изосить что либо в оном влагалище. А в финале выяснится это это была демоверсия.
Аноним 27/11/22 Вск 17:41:57 568420 125
>>568418
>Я же спрашиваю про прикладную науку.
А я тебе про что по-твоему?
Аноним 27/11/22 Вск 18:05:33 568421 126
>>568420
Про модель влагалища. А мне нужен научный вердикт по эмпирике.
Аноним 27/11/22 Вск 18:39:49 568422 127
>>568421
А откуда ты будешь знать что исследование честное, и что вообще было какое-то исследование а не то что тебе просто сказали заведомо не верный результат чтобы обмануть без всяких иследований?
Аноним 27/11/22 Вск 19:25:27 568424 128
>>568422
Посмотрю на фамилии авторов. Если там жиды - однозначно наёб. Это научный метод, азы.
Аноним 27/11/22 Вск 19:50:34 568427 129
>>567887 (OP)
Как разорвать ткань пространства-времени локально, скажем, для передачи только одного канала связи

например


я хочу походить по марсу или фобосу(сптнк марса) в аватаре(гуманодиной формы робот, управляется из костюма на земле в котором сидит оператор(Я)).
проблема щас в том, что задержка сигнала между З и М - от 3 до 24 минут(в зав-ти от позиции планет).

как убрать этот лаг? до 50мс-1сек максимум.
только разорвав т.н. пространство-время?
Аноним 27/11/22 Вск 20:00:01 568429 130
>>568427
Берёшь материю с отрицательной массой и рвешь. Просто.
Аноним 28/11/22 Пнд 00:25:52 568437 131
Какие минусы у автомобилей на водороде? Я знаю только один, что вроде бы как в теории они бак может взорваться если его разгерметизировать.
Аноним 28/11/22 Пнд 08:56:48 568442 132
>>568427
Дурачёчек ты. Если разорвать пространство-время, то никакой сигнал не пройдёт от слова совсем. Сигналы только по сплошному пространству-времени передаются. Разрывы для них - непреодолимое препятствие.
Аноним 28/11/22 Пнд 08:58:26 568443 133
>>568437
Водород протекает сквозь кристаллическую решётку металла и сквозь любые микрощёлочки и микротрещинки. Бензин так не протекает, т.к. у бензина молекулы большие и жирные.
Аноним 28/11/22 Пнд 08:59:32 568444 134
>>568284
Ты видать не смотрел советские кукольные мультики. Вкатывайся в мультики, там есть всё.
Аноним 28/11/22 Пнд 09:00:51 568445 135
Аноним 28/11/22 Пнд 09:03:57 568446 136
Аноним 28/11/22 Пнд 09:08:16 568448 137
image.png 244Кб, 860x500
860x500
>>568416
Не изнашивается, но растягивается.
Мой хуй идеальный детектор, которым я определяю, рожала баба или нет, любит ли баба хуи побольше, или ей и маленькие норм.
Отслеживаю один из пиздо-объектов на протяжении 10 лет. С 15 до 20 лет у объекта была очень узкая тугая писечка. Потом повадилась ебаться с гигачэдами и теперь в пизда как рукав.
Аноним 28/11/22 Пнд 09:51:41 568451 138
Какой биологический смысол в фетишах и прочих девиациях?
Ну, допустим гомосексуальность это какой-то сбой в идентификации объекта.
Какакие-то отклонения, связанные с агрессией и болью - тоже чётотам перемыкает в мозгах, хуё-моё, центры близко, доминирование и подчинение присутствуют в половом поведении всех достаточно сложных животных, хотя бы чисто по биомеханическим причинам.
Но как объяснить футфетиш? Влечение к ампутантам? Это ведь не извращения, приобретаемые во взрослой жизни, это явно что-то прошитое по крайней мере с раннего детства. Каннибальские наклонности так проявляются?
Аноним 28/11/22 Пнд 10:07:15 568452 139
>>568442
Там мы его разорвем в двух местах, а потом их склеим, и все.
Аноним 28/11/22 Пнд 12:11:29 568455 140
>>568446
А если тепло дома. Я не потеряю сознание, если буду весь день так дышать?
Аноним 28/11/22 Пнд 14:07:13 568462 141
>>568455
Сердце начнёт плохо работать.
Аноним 28/11/22 Пнд 14:16:18 568464 142
>>568437
Главный минус в отсутствие отработанной технологии по производству, хранению и использование водорода.
Делать ДВС на водороде бессмысленно, ибо проигрывает по всем пунктам метану.
Топливные элементы сейчас по сути штучный продукт и слишком дорогие из-за низкого ресурса.
Водород до сих пор ебашут из метана с большими потерями энергии. По сути мы отложено сжигаем метан.
Аноним 28/11/22 Пнд 14:45:18 568466 143
А правда, что антигрипин это грипин из анти вещества?
Аноним 28/11/22 Пнд 17:22:27 568475 144
>>568466
Конечно.
А антисемит - это жыд из параллельного антимира.
Аноним 28/11/22 Пнд 20:33:15 568502 145
Аноним 28/11/22 Пнд 21:48:42 568506 146
Посоветовали такое видева: https://youtu.be/03OsXsgvt8I
Просто может кто смотрел или знаком с этим Дойчем и его книгой "Структура реальности..." Я так понял, там очередная лапша про интерференцию щелей в параллельной вселенной мертвого кота.
Аноним 28/11/22 Пнд 23:05:05 568516 147
Как ОСТРОВ может двигаться нахуй по 200 метров в год? Он же не плавает по океану, а закреплён на земле? У него что, своя мини-тектоническая плита нахуй персональная?

>Остров знаменит как «Северное кладбище Атлантики» (наряду с «Южным Кладбищем Атлантики» в районе мыса Хаттерас), так как только после 1800 года около него было зарегистрировано около 350 кораблекрушений. Навигация в прибрежных водах острова осложнена наличием двух встречных течений: тёплого Гольфстрима и холодного Лабрадорского течения, порождающих водовороты. Также благодаря этим течениям остров движется со скоростью 200 м/год, что тоже может быть причиной навигационных ошибок[2

https://ru.wikipedia.org/wiki/Сейбл
Аноним 29/11/22 Втр 00:22:10 568523 148
>>568516
>Как ОСТРОВ может двигаться нахуй по 200 метров в год?
С одной стороны размывается, а с другой нарастает за счёт новых отложений грунта.
В Арктике полно таких островов, в т. ч. и у нас. Береговая линия в районах вечной мерзлоты нестабильна.
Аноним 29/11/22 Втр 00:23:12 568525 149
>>568464
А разве электролизом водород не получают?
Аноним 29/11/22 Втр 08:27:30 568530 150
>>568525
Электролиз на порядки дороже чем конверсия метана.
Аноним 29/11/22 Втр 08:40:07 568533 151
>>568506
Правильно понял.
Аноним 29/11/22 Втр 10:16:08 568538 152
Аноним 29/11/22 Втр 11:37:44 568541 153
>>568538
> :3
Но у Дойча кстати довольно оригинальная лапша, где он через интерференцию щелей обосновывает наличие параллельных миров. Не особо научно, но на философско-гуманитарном блабла вполне занятное чтиво.
29/11/22 Втр 12:57:25 568547 154
>>568541
>занятное чтиво
это бред
Аноним 29/11/22 Втр 16:11:38 568552 155
>>568530
И чем же дороже, если тратится только электричество?
Аноним 29/11/22 Втр 17:59:34 568556 156
>>568552

Паровая конверсия — получение чистого водорода из лёгких углеводородов (например метана, пропан-бутановой фракции) путём парового риформинга (каталитической конверсии углеводородов в присутствии водяного пара). Реформирование газового пара является самым популярным и самым дешевым способом производства водорода. По сравнению с, например, электролизом воды, количество водорода, полученного на единицу потребляемой энергии, намного выше.
Аноним 29/11/22 Втр 19:53:12 568558 157
Kinesinwalking.gif 108Кб, 380x286
380x286
https://ru.wikipedia.org/wiki/Кинезин
Как эта хуйня понимает куда ей идти и для чего? Она осознает себя, или там всё на инстинктах? Или это сугубо физический механизм?
Аноним 29/11/22 Втр 21:11:25 568560 158
>>568558
Сугубо физический механизм. Буквально наномашина. Она движется только вперед по нужному направлению, по которому закручивается микротрубочка, пока у димера кинезина есть приход топлива(АТФ) или когда его не сломает другой специальный белок вахтер на конечной точке.
Аноним 29/11/22 Втр 23:20:48 568564 159
изображение.png 3703Кб, 1700x2500
1700x2500
Правда ли что каждая клетка рано или поздно отмирает и заменяется новой? Таким образом через Х лет\месяцев в человеке все клетки являются не теми, что были раньше?
Аноним 30/11/22 Срд 00:29:06 568567 160
Подскажите, пожалуйста, ответ на глупый вопрос. Вот Бойль делал такой эксперимент, он брал установку Торричелли (трубку полную ртути, запаянную с одного конца и перевернутую затем в блюдо с ртутью) и помещал в большую емкость, к которой снизу был подсоединен насос, а сверху емкость более-менее герметично закрывалась тоже. При этом уровень ртути в трубке не уменьшался после того как установка засовывалась в емкость, а начинал этот уровень уменьшаться только после откачки воздуха из емкости.

Вопрос: почему уровень не понизился сразу, когда только трубку с блюдом помещали в емкость и закрывали там? Т.е. почему атмосферное давление оставалось внутри емкости, даже когда она уже была изолирована?
Аноним 30/11/22 Срд 00:36:15 568568 161
>>568564
нейроны
стволовые клетки
Аноним 30/11/22 Срд 00:58:01 568570 162
>>568567
Сначала воздуху не давал разлетаться окружающий воздух, потом вместо него пришла стенка.
Аноним 30/11/22 Срд 03:04:36 568574 163
>>568568
Что нейроны? Что стволовые клетки? Это какой-то пук или ты чат-бота тестируешь тут?
Аноним 30/11/22 Срд 03:22:02 568575 164
Можно ли ловить карбон из воздуха и делать из него волокно?
Аноним 30/11/22 Срд 03:50:25 568576 165
image.png 439Кб, 800x558
800x558
>>568575
Можно. При том технология отработана и применяется людьми на планете Земля уже несколько тысячелетий.
Но зачем?
Аноним 30/11/22 Срд 03:51:54 568577 166
>>568564
Нет, не правда.
Многие клетки делятся на двое вместо отмирания.
Аноним 30/11/22 Срд 03:58:54 568578 167
>>568455
Ну начнём с того, что у тебя пересохнет рот. Язык опухнет, гланды начнут болеть из-за высыхания.
Закончим тем, что рано или поздно ты устанешь дышать часто, у тебя начнёт болеть диафрагма, пойдут спазмы бронхов от усталости и пересыхания. В коне концов ты сдашься и в твоём мозгу включится защитная реакция, от которой неожиданно возникнет мысль, что ты долбоёб, занимаешься хуйнёй и лучше бы сделал домашку.
Аноним 30/11/22 Срд 04:03:24 568579 168
>>568451
Пока омежка дрочит на фетиши, гигачэд оплодотворяет тяночек своими хорошими генами. Результат: и гигачэд и омежка заняты делом и не бугуртят. Оба приносят пользу популяции или хотя бы семье. Гигачэд приносит пользу путём распространения полезных гигачэдовских генов. А омежка тем, что въёбывает на заводе и содержит блядей, оплодотворённых гигачедом, и гигачедовских выблядков, когда женится на РСП.
Аноним 30/11/22 Срд 05:53:50 568580 169
>>568574
Срок жизни человеческих нейронов неизвестен и они не делятся. Стволовые клетки только делятся и не умирают от времени.
Аноним 30/11/22 Срд 07:03:40 568582 170
>>568558
Как механизм храповика понимает, что может только в одну сторону крутиться?
Аноним 30/11/22 Срд 07:33:46 568583 171
Аноним 30/11/22 Срд 19:27:36 568594 172
>>568502
Мне кажется потому что механизмы частоты дыхания и сердцебиения довольно жостко завязаны.
И на "прошытом" уровне, т.е. регуляция скорости одного тупо по скорости другого, так и физически, например слишком часто дышишь но при этом активно не двишаешься-пересыщаешь кровь кислородом.
Это как с оборотами двигателя и газом, подал больше топлива-значит возросли обороты, значит нагрузка должна быть выше. А если увеличить подачу топлива без увеличиения оборотов и без увеличения нагрузки (как-то, хоть это и завязано просто на уровне протекания процесса) то двигатель по пизде пойдёт.

Я пробовал, если даже не очень долго часто дышать, при этом сидеть спокойно, не бегать, упражнения не делать-появляется неприятное ощущение в сердце.
Попробуй, может у тебя по-другому.
Аноним 30/11/22 Срд 22:16:53 568597 173
Аноним 30/11/22 Срд 22:23:43 568598 174
>>568580
Ну а клетки кожи? Они же отмирают? Тогда почему татуировки на всю жизнь остаются, а не просто удаляются сами когда клетки отмирают.
Аноним 30/11/22 Срд 22:30:50 568599 175
>>567887 (OP)
А бывают ли двуполушарные ии?
Аноним 01/12/22 Чтв 02:39:50 568607 176
>>568599
Да, бывают. Один из них, например, задал этот >>568599 вопрос.
Аноним 01/12/22 Чтв 02:41:01 568608 177
>>568598
Ты представь себе, что половина твоих клеток кожи уже мертва и не удалились из организма. Охуеть, не так ли?
Аноним 01/12/22 Чтв 07:20:53 568609 178
>>568608
Не, там краску походу живые макрофаги жрут. И красятся. А она не переваривается и остается для следующего поколения макрофагов.
Аноним 01/12/22 Чтв 16:15:27 568617 179
>>567887 (OP)
А что если масса человека увеличится в 1000 раз? Допустим с 75 кг до 75 тонн, что с ним будет происходить на земле?
Аноним 01/12/22 Чтв 20:27:53 568622 180
>>568617
Он будет гнить. Разложится до чистого скелета за несколько недель или месяцев.
Аноним 01/12/22 Чтв 20:58:23 568626 181
>>568622
То есть сразу после увеличения массы начнёт гнить? Даже пожить не успеет?
Аноним 01/12/22 Чтв 20:59:29 568627 182
>>568626
>То есть сразу после увеличения массы умрёт и начнёт гнить? Даже пожить не успеет?
Быстрофикс
Аноним 01/12/22 Чтв 23:32:27 568630 183
>>568626
>Даже пожить не успеет?
До смерти мозга пройдет минут пять-шесть.
>сразу после увеличения массы начнёт гнить?
Ранние трупные явления начинаются через несколько часов после смерти, поздние - через несколько суток.
Аноним 02/12/22 Птн 03:34:01 568631 184
ДНК открыли и раздуплились зачем она только в конце 19 века. Как до этого люди считали инфа передавалась от родителя к ребенку? С помощью бога?
Аноним 02/12/22 Птн 06:51:38 568633 185
Аноним 02/12/22 Птн 07:26:31 568634 186
300 лет назад для совершения фундаментального научного открытия достаточно было скинуть два шара с Пизанской башни, а один человек мог знать все открытые на тот момент естественнонаучные законы.
В наше время для проверки незначительных гипотез нужно строить БАК, а у каждого учёного своя глубокая специализация.
Совершать открытия всё сложнее, мы асимптотически приближаемся к потолку развития человечества.
Так откуда же вылезла концепция технологической сингулярности? Какие вообще есть основания утверждать, что нас ждёт нечто подобное?
Аноним 02/12/22 Птн 09:00:37 568637 187
>>568634
Никаких оснований нет.
Это гипотезы долбоебов, которые боятся смерти и мечтают о рае.
Лысые шимпанзе не меняются. И не учатся, что еще печальнее.
Можно спустить обезьяну с ветки, но хватательный рефлекс у неё не исчезнет.
Все разговоры о бессмертии в любой его форме - это попытка "ухватить" побольше времени для стимулирования центров удовольствия в межушном ганглии. Они все обречены на провал из-за фундаментальных законов мироздания.
Аноним 02/12/22 Птн 10:13:20 568645 188
>>568634
Сингулярность это псевдонаучная религия. Основные факты уровня я так хочу.
1) Никто не знает что такое сознание и как работает человеческий разум.
2) Потенциал ИИ как минимум раздут.
3) Нет никаких фактов доказывающих что человеческое сознание можно отцифровать.(смотри пункт 1)
Вывод: сингулярность это очередная шняга которой университеты в США отжимают донатики у религиозных организаций. Остальное каргокульт.
Аноним 02/12/22 Птн 11:31:24 568649 189
>>568634
У нас инструменты улучшаются, ты компьютером можешь сейчас делать дохуя, что раньше бы не смог. Грузишь в комп фотографию неба в разрешении терапиксельном (или какое у них там охуенное я не знаю), а тебе нейросеть нужные классы звезд и аномалии интересные сама ищет.
Аноним 02/12/22 Птн 14:23:04 568654 190
>>568631
С помощью кончи
Ты совсем девственник-аутист?
Аноним 02/12/22 Птн 15:00:03 568655 191
>>568631
> только в конце 19 века
А разве не после Второй мировой, в начале 50-х годов..?
Аноним 02/12/22 Птн 15:04:07 568656 192
ntc
Аноним 02/12/22 Птн 17:05:41 568660 193
>>567887 (OP)
Где найти научную литературу про интеллект мышей, про то как их дрессируют, обучают?
Лучше скиньте ссылки.
Аноним 02/12/22 Птн 17:06:35 568661 194
>>568655
После открытия клеток, вопрос был только в выборе правильного компонента.
Аноним 02/12/22 Птн 20:38:18 568667 195
>>568630
А если эти 75 тонн достигаются не за счёт увеличения массы, а за счёт плотности, без изменения объема тела?
Аноним 02/12/22 Птн 21:24:59 568668 196
>>568631
Практически они знали. Поймали свиней заперли их загон и смотрят что дальше будет. Свиньи одомашнились, а лисы нет. У свиней рождаются свиньи а у коров коровы. А свинекоровы совсем не рождаются
На примере огня тебе не надо знать что горение это экзотермическая химическая реакция в результате которой выжеляется угликислый газ. Ты просто берешь спички и делаешь костер.
Аноним 02/12/22 Птн 21:30:36 568669 197
>>568667
То он станет марвелодебилом как ты
Аноним 02/12/22 Птн 21:57:48 568670 198
>>568668
>свинекоровы совсем не рождаются
Иногда рождаются. Эксперименты по скрещиванию людей с животными не на пустом месте начались.
Аноним 02/12/22 Птн 22:25:38 568672 199
16643058495760.jpg 193Кб, 755x755
755x755
>>568669
Я попрошу, анимедебилом.
Аноним 02/12/22 Птн 22:32:01 568674 200
>>568670
Гибриды людей с собаками свиньями и мартышками получаются и так, просто из людей.
Благодаря социализму и толерастии путём случайных мутаций может получаться что угодно, главное чтобы снаружи оно было как человек, тогда он по закону социализма и толерастии признаётся человеком и ему сохраняется жизнь, причём даже помогается жить даже, и кроме этого, плодиться помогается, если не жизнеспособен. При этом совершенно необязательно чтобы оно и внутри было человеком. Достаточно только внешнего сходства, снаружи.
Тут просто из логики случайных мутаций будут появляться ужасные мутанты и гибриды, единственные два критерия:внешне должен иметь вид человека и хорошо плодиться.
Аноним 02/12/22 Птн 22:35:00 568675 201
1670009699631.jpg 36Кб, 600x344
600x344
Аноним 02/12/22 Птн 23:01:55 568676 202
>>568674
Из случайных мутаций у тебя только старикашка получится. Для свинособаки, нужно заставить все свои клетки жить в свинособачьих условиях. Чтобы они проходили отбор и изменялись в заданном направлении.
Аноним 02/12/22 Птн 23:23:35 568677 203
>>568676
>Чтобы они проходили отбор и изменялись в заданном направлении.
Это чтобы 100% особей такими были, а при многомиллиардной выборке такие появляются в огромном количестве и без отбора средой.
Аноним 02/12/22 Птн 23:25:21 568678 204
>>568676
>клетки жить в свинособачьих условиях.
>>568674
>социализма и толерастии
Ну собственно...
Аноним 03/12/22 Суб 13:41:59 568686 205
>>567887 (OP)
Извиняюсь за тематический вопрос не совсем по адресу, но нужно мнение экспертов в информационной сфере.
С каким кодеаом, форматом и контейнеров можно закодировать фильм так, чтобы он не требовал почти никаких ресурвом пк при воспроизведении на обычных программных плеерах по типу vlc или mpc hc? Какие кодеки требуют минимальных ресурсов ПК при разрешении видео меньше HD-ready, с обычным звуком 5.1?
Нужно для фонового воспроизведения при работе в тяжёлых приложениях на одном компе.
Аноним 03/12/22 Суб 14:07:04 568687 206
>>567927
Помню игру, которая в случае проигрыша стирает рандомный файл на харде. Можно ли сделать игру, которая в случае проигрыша сжигает процессор? Есть ли такие доступы из операционной системы?
Аноним 03/12/22 Суб 14:13:27 568688 207
>>568541
В некоторых главах он откровенно шизит (проталкивая своё видение мультивёрса, просто потому что ЯСКОЗАЛ), но, на самом деле, "Структура реальности" очень познавательное чтиво.
Аноним 03/12/22 Суб 14:29:14 568689 208
>>568686
Тебе в /s/, по железкам в /hw/
Аноним 03/12/22 Суб 14:56:58 568691 209
Ученые-под-хвост-толчёные, ответьте, когда мне стоит ждать в продаже хоть сколько-то функциональных человекоподобных роботянок с искусственным интеллектом? Доживу ли я вообще?
Вопрос не о том, смогу ли я позволить себе такую покупку и не о ее целесообразности.
Аноним 03/12/22 Суб 15:08:40 568694 210
>>568691
>Вопрос не о том, смогу ли я позволить себе такую покупку и не о ее целесообразности.
Нет, в этом и вопрос, т.к. если мог бы позволить то уже давно бы сделали.
Аноним 03/12/22 Суб 15:29:59 568695 211
>>568691
Роботянки(персонажи) были в компьютерных играх 90-х. Чем все закончилось? Женщины обьясняют теперь разработчикам как роботянки должны выглядить и вести себя с игроками.
Все что тебе надо знать о будующих роботянках.
Аноним 03/12/22 Суб 15:51:14 568696 212
>>568695
++
>Все что тебе надо знать о будующих роботянках.
Скорее о его будущем, куколд-хуесос опущенный всегда будет куколдом-хуесосом опущенным. Причина и следствие всегда сильно связаны, следствия не изменятся если причина не изменилась.
Аноним 03/12/22 Суб 16:18:39 568698 213
>>568694
>>568695
Долбоебы, я вас спрашиваю про возможность технически реализовать такого робота и про то, как скоро после первых прототипов все будет готово для их серийного выпуска. У меня 15 млн рублей лежит в виде бабкиной трешки в москве, а в цивилизованном мире такие деньги вполне себе реально заработать среднему спецалисту. Так что вопрос о стоимости не стоит, как я уже написал выше, специально для конченых ебланов.
Аноним 03/12/22 Суб 16:41:40 568701 214
>>568689
Знаю, но вдруг появится наукоанон, интересующийся этим с такой стороны. Информатика же тоже наука. Спасибо в общем, там тоже задавал свой ответ.
Аноним 03/12/22 Суб 17:17:52 568702 215
>>568698
>вопрос о стоимости не стоит
Во-первых, ты нищий, во-вторых, опущенный.
Т.е. тебе ни в каком случае не светят такие штуки.
Аноним 03/12/22 Суб 21:51:39 568708 216
>>568702
Завистливой пидорашке ниприятна
Аноним 04/12/22 Вск 00:17:50 568714 217
>>568708
А тебе приятно, когда ссут в ебало?
Аноним 04/12/22 Вск 11:57:49 568718 218
Аральское море.jpg 94Кб, 704x437
704x437
>>567887 (OP)
Какие прогнозы по высыханию Арала? В северной части будут расширять плотину и уровень воды вырастет. А вот что насчёт южной? Будет ли продолжить высыхать оставшаяся узкая полоса воды на западе? Высохнет ли окончательно восточная часть, превратившаяся в пересыхающее озеро?
Аноним 04/12/22 Вск 13:16:48 568720 219
>>568708
Что неприятно? Что ты такой как выше написано?
Аноним 04/12/22 Вск 15:03:35 568725 220
Как сейчас биологи манипулируют с ДНК/РНК/другими клеточными хуйнями на предмет перемещения отдельных молекулярных кусков?

Через 100500 каких-нибудь промежуточных буферных растворов? У нас-же ведь нет пока каких-нибудь нанопинцетов, так?
Аноним 04/12/22 Вск 15:29:00 568727 221
>>568725
>Через 100500 каких-нибудь промежуточных буферных растворов? У нас-же ведь нет пока каких-нибудь нанопинцетов, так?
Однозначно есть, например ибм делала картинки атомные где толщина линии один атом
Но целесообразность использования в биологии неясная, наверное слишком долго такое делать
Аноним 04/12/22 Вск 22:19:15 568735 222
>>567887 (OP)
Реквестирую что-нибудь по теории струн
У меня хорошая база в математике, но с физикой не сложилось...
Аноним 05/12/22 Пнд 02:46:11 568741 223
Десять-пятнадцать лет назад все учоные в говне мочоные активно заявляли, что УЖЕ ВОТ СЕЙЧАС НАЧНЕМ КЛОНИРОВАТЬ И ВОЗВРАЩАТЬ ИСЧЕЗНУВШИЕ ВИДЫ, СКОРО МАМОНТ!!!111 В итоге напиздели с три короба, загребли бабок за расшифровку днк, а воз и ныне там. В чем проблема, кроме жадности?
Аноним 05/12/22 Пнд 04:26:13 568743 224
>>568741
А в чем там сложность-то, древняя ДНК с современными органеллами не совмещается?
Аноним 05/12/22 Пнд 06:59:00 568747 225
>>568741
>В чем проблема, кроме жадности?
В том, что мамонты нахуй никому не нужны.
Аноним 05/12/22 Пнд 07:00:51 568748 226
>>568718
Да нахуй он нужен, орал этот? Там же всё давно здохло, одни лиш бактерии-экстремофилы остались
Аноним 05/12/22 Пнд 07:06:31 568749 227
image.png 172Кб, 664x875
664x875
>>568691
Ну вот смотри. В этой пятилетке в массы внедрились голосовые чатботы типа яндекс-алисы. И китайцы точно таких же чатботов встраивают в кукол. Вот прям сразу.
Точно так же и с искуственным интеллектом будет как только так сразу.
Что касается функциональности, то у них уже есть самоподогрев, конвульсии пизды, симуляция дыхательных звуков. Какого функционала тебе тогда не хватает?
Аноним 05/12/22 Пнд 09:00:31 568756 228
>>568698
>такого робота
Какого?
Ты хотя бы пример приведи или эскиз модели.
Если ты хочешь какую-нибудь Аэлиту, то ты не доживешь.
Интеллекту любому прежде всего нужен источник энергии.
Пока что даже суперкомпьютеры, питаемые ядерными реакторами сосут по производительности серого вещества в коробке.
Аноним 05/12/22 Пнд 09:02:07 568757 229
>>568741
>В чем проблема, кроме жадности?
Эксперименты над клонами считаются не этичными и запрещены.
Потому что сегодня ты мамонта клонируешь, а завтра - резиновую попу.
Аноним 05/12/22 Пнд 10:18:48 568761 230
>>567209 →
>Ну понятно что не влоб продолжать следовать старой модели, я скорее про само название. Современная физика утверждает что эфира как всепроникающей среды не существует и что э/м волны распространяются как бы сами по себе. Но ведь это пиздежь - волны распространяются как раз в среде пространства-времени, тоже всепроникающей среде но с другим названием и свойствами.
Можно поднять эту тему про фотоны?
Аноним 05/12/22 Пнд 10:20:25 568762 231
Давайте рассмотрим среду из плотноприжатых фотонов - то есть фотоны собственно никуда не летят, а лишь передают импульс со скоростью света. Конечный фотон в цепочке проявляет себя как частица с энергией.
Аноним 05/12/22 Пнд 10:48:44 568765 232
>>567191 →
>Эфир же слишком механистическая модель, в том смысле в нем нельзя делать обобщения движения. Всякие лагранжианы выглядит как уродливое нагромождение.
Если ты попытаешь для этого как-то пригладить эфир, то у тебя получатся кривая модель пространства-времени, где придется отказывается от механической модели, а это сродни отвергания самого эфира.
А какой эфир? Что вообще подразумевается под эфиром? Какая модель?
Если взять ту же среду из плотноприжатых фотонов, то для движения тела фотоны должны перетекать с фронта движения в тыл. Со скоростью взаимодействия между фотонами, которая равна скорости распространения импульса через плотноприжатые фотоны и равна скорости света. Поэтому когда скорость тела начнет приближаться к скорости света - то фотоны не будут успевать с тем же успехом перебегать с фронта движения в тыл - здесь и всплывают "Всякие лагранжианы".
Аноним 05/12/22 Пнд 13:00:14 568767 233
>>567887 (OP)
Анон, смотри, что если тройня, рождённая в результате искусственного оплодотворения, покажет на ДНК-тесте,
что матерью является целочка - мне дадут откос от службы, или нет?
Аноним 05/12/22 Пнд 13:31:09 568771 234
>>568761
> Можно поднять эту тему про фотоны?
Что именно поднять?
Аноним 05/12/22 Пнд 13:37:29 568772 235
>>568762
> Давайте рассмотрим среду из плотноприжатых фотонов - то есть фотоны собственно никуда не летят, а лишь передают импульс со скоростью света. Конечный фотон в цепочке проявляет себя как частица с энергией.
Что ещё за плотноприжатые фотоны? Фотон это кусок электромагнитного поля, как этот кусок можно куда то прижать?
Аноним 05/12/22 Пнд 14:51:54 568774 236
>>568772
>Что ещё за плотноприжатые фотоны?
Да то это местный шиз претендент на нобелевку.
Аноним 05/12/22 Пнд 15:46:43 568777 237
>>568772
> Фотон это кусок электромагнитного поля
То есть в твоем представлении фотон физически не существует? Фотонов нет? Ты шиз?
Аноним 05/12/22 Пнд 17:36:47 568799 238
>>568777
> > Фотон это кусок электромагнитного поля
> То есть в твоем представлении фотон физически не существует? Фотонов нет? Ты шиз?
Это было что то из раздела "сам пошутил, сам посмеялся".
Сам придумал что то про меня, сам опроверг.
Аноним 05/12/22 Пнд 17:56:36 568801 239
>>568799
Не веди себя как мендочка, не нужно здесь этих виляний - фотон, в твоем представлении, физически существует или нет?
>кусок электромагнитного поля
Речь про физическую среду, которая и порождает эм поле.
Аноним 05/12/22 Пнд 20:29:04 568811 240
>>568801
Фотон существует в том же смысле, в каком существует капля воды. Капля воды это просто часть воды, фотон аналогично просто часть э/м поля.
Аноним 05/12/22 Пнд 21:07:28 568815 241
>>568811
ты путаешь причину и следствие - среда порождает поле, а не наоборот)
Аноним 05/12/22 Пнд 21:37:52 568816 242
>>568815
Среды не существует, это пустое слово, есть совокупность полей.
Аноним 05/12/22 Пнд 22:22:15 568817 243
>>568816
>совокупность полей
футбольных?
Аноним 05/12/22 Пнд 23:19:00 568819 244
>>568817
Ты тупой? Сказано же совокупность полей, т.е. разных
Аноним 06/12/22 Втр 06:37:08 568822 245
>>568756
Облачные вычисления?
Аноним 06/12/22 Втр 07:46:28 568825 246
>>568815
Это было заблуждение, мы пытались эмпирический опыт распространить на микро-мир.
Аноним 06/12/22 Втр 08:41:01 568829 247
>>568819
волейбольное ещё что-ли? )
Аноним 06/12/22 Втр 08:44:32 568830 248
>>568825
>мы пытались эмпирический опыт распространить на микро-мир.
А зачем? Чувственным восприятием микро-мир не понять. Приборами тоже - только дедуктивной логикой.
Так вот следуя дедуктивной логике - полей без среды, которая порождает эти поля - не существует.
Аноним 06/12/22 Втр 12:34:03 568839 249
>>568829
Как ты слаб в теории поля
А теннисное? Как ты без тенисного будешь теорию сводить? А баскетбольное?
Аноним 06/12/22 Втр 12:36:53 568841 250
>>568830
>полей без среды, которая порождает эти поля
Докажи, пустой пиздёж какой-то.
Аноним 06/12/22 Втр 12:46:59 568842 251
>>568841
Доказать нельзя, ибо
>Чувственным восприятием микро-мир не понять. Приборами тоже
Можно только прочувствовать применив больные фантазии дедуктивню логику. И не забывай прижимать фотоны!
Аноним 06/12/22 Втр 13:16:08 568845 252
>>568842
Так у тебя какой-то пустой пиздёж в квадрате
Никакая "среда" не требуется, ты просто выдумал слово которое ничего не значит, и утверждаешь что оно порождает поля.
Аноним 06/12/22 Втр 13:25:11 568847 253
>>568845
Перетолстил что-ли? Я не он.
Аноним 06/12/22 Втр 13:39:50 568849 254
>>568845
>оно порождает поля
что такое поле?
Некий объем пространства с измененными физическими параметрами среды относительно состояния среды в остальных точках пространства.
Аноним 06/12/22 Втр 15:29:21 568850 255
>>568849
Поле это только гладкая функция от координат и времени, больше ничего сущностного полю не нужно. Многие ФИЗИЧЕСКИЕ свойства поля вылезают из геометрии или особенностей системы координат. В более общем смысле поле это вообще расслоенное многообразие - этакий многомерный геометрический выверт.
Аноним 06/12/22 Втр 16:15:13 568851 256
>>568850
Пространство без материи, без материальной среды не существует. Размерностью может обладать только материальный объект. "НИЧТО" не может обладать размерностью - ибо там ничего нет, а раз ничего нет, то "НИЧЕГО" и не существует.
Трехмерную сетку пространство можно наложить только на материю, на материальную среду без пустоты.
Поэтому ты опять путаешь причину и следствие, заявляя о привязке поля к пространству - твоя трехмерная сетка пространства виртуальная. Она не способна существовать без материи.
Аноним 07/12/22 Срд 09:05:03 568869 257
Аноним 07/12/22 Срд 20:43:46 568893 258
>>568851
Демагогия, уровня "атомов не существует".
Аноним 07/12/22 Срд 20:47:20 568894 259
>>568883
Всё сложнее, научпоперы конечно же не правы, но и вульгарное "детектор влияет на частицу" тоже не совсем правда, т.к. есть интересные эксперименты, где детектор вовлекается в суперпозицию.

Найди себе время, почитай нормальные учебники, тебе надо понять, как работает алгебра наблюдаемых (это такая специальный маленький раздел математической физики, которую могут понять даже девятиклассники, и которую используют для описания квантовой механики).
Аноним 07/12/22 Срд 22:27:11 568895 260
>>568883
>это действительно так, или все эти уебаны просто не делают важную оговорку
второе
Аноним 08/12/22 Чтв 03:02:43 568911 261
>>567887 (OP)
Реквестирую лучший бесшумный бензиновый генератор для квартиры, по критерию цена-качество.
Аноним 08/12/22 Чтв 10:40:57 568912 262
image.png 414Кб, 1068x683
1068x683
>>568911
Вот этот. Нагенерирует тебе бензина хоть купайся. Правда вонять будет. Так что поищи ещё и хорошую вытяжку.
Аноним 08/12/22 Чтв 10:41:52 568913 263
image.png 1686Кб, 978x1608
978x1608
>>568883
>эксперимент с двумя щелями
Аноним 08/12/22 Чтв 11:00:55 568915 264
>>568913
Ну и в чём разница - что меняется, когда добавляется наблюдатель?
Аноним 08/12/22 Чтв 11:07:11 568916 265
>>568915
> Ну и в чём разница - что меняется, когда добавляется наблюдатель?
Результат меняется.
Мимо
Аноним 08/12/22 Чтв 11:15:17 568917 266
>>568741
1. Мы не можем воспроизвести того самого мамонта. Причин для этого несколько:
1.1. У нас нет пробирки, для полного цикла выращивания такого организма. Следовательно, надо из пробирки встраивать куда-то материал, для прохождения последующих этапов развития, и рождения.
1.2. Полученного материала днк попросту недостаточно.
1.3. Еще ворох проблем.
2. Получится уже не мамонт, а НЕХЪ, выращенная на базе современного слона.

Опустим все мучения и расходы, которые пришлось бы понести. Возникает вопрос: а зачем? Зачем нам этот новиоп. Нет, сделать его по приколу, наверняка бы стоило. Но это говно без задач. На которое никто не даст квинтиллион зелени, даже если все будет готово.
Аноним 08/12/22 Чтв 11:23:45 568918 267
>>568917
> 1.1. У нас нет пробирки, для полного цикла выращивания такого организма.

В пизду слонихе запихаем.
Аноним 08/12/22 Чтв 11:28:43 568919 268
>>568917
>Возникает вопрос: а зачем?
Разводить в Сибири и жрать
Аноним 08/12/22 Чтв 12:08:42 568922 269
>>568918
Вот тут мы и получили новиопа. А не белого серого расово верного арийского мамонта. На самом деле мы получили его еще раньше, но это уже не так важно.
>>568919
> >Возникает вопрос: а зачем?
> Разводить в Сибири и жрать
Биом не торт. И не окупится даже в виде ферм. Зачем пробирить говядину на пенсии, если есть обычная говядина.
Аноним 08/12/22 Чтв 12:25:33 568923 270
Нужна ли школьная программа физики для понимания квантовой механики? Если да, то насколько? Какие учебники есть?
Аноним 08/12/22 Чтв 12:48:34 568924 271
>>568917
>Полученного материала днк попросту недостаточно
Так, Я не понял, у них есть днк мамонта или нет? Потому что если нет, то воскрешателей нужно просто признать мошенниками и перестать говорить о них.
Аноним 08/12/22 Чтв 12:53:32 568925 272
>>568923
Нужно, но школьная программа это только обзорный ознакомительный курс. Без нее ты не вкатишься в базовые основы физики.

Для хорошего понимания квантовой физики нужно уже минимум знать гамильтонов формализм, что входит не во все вузовские курсы.
Аноним 08/12/22 Чтв 12:59:08 568926 273
>>568924
Есть кнш. А о воскрешателях имхо, вообще бесполезно говорить. Вряд ли будут пробовать всерьёз.
Аноним 08/12/22 Чтв 13:00:02 568927 274
>>568925
Так зачем она нужна если квантовая механика не связана с классической? Я, кстати, не про математику а про физику конкретно.
Аноним 08/12/22 Чтв 13:02:46 568928 275
>>568927
Это как пытаться построить многоэтажный дом, без умения лепить обычные куличики.
Аноним 08/12/22 Чтв 13:40:30 568929 276
>>568928
Это из разряда "если я ниасилил то то не смогу и это"? Есть ли конкретная связь? Куличики на стройке как-то не лепят.
Аноним 08/12/22 Чтв 13:42:25 568930 277
>>568869
Нелинейные динамические системы детерминированы, но непредсказуемы.
Читать про хаос Глейка.
Аноним 08/12/22 Чтв 14:00:12 568931 278
>>568923
> Нужна ли школьная программа физики для понимания квантовой механики? Если да, то насколько? Какие учебники есть?
Если ты осилишь сразу вузовский учебник, то не нужна.
Аноним 08/12/22 Чтв 14:10:55 568932 279
>>568931
Так я и спрашиваю. Требуется ли она для понимания или нет, используется ли. Зачем учить просто так то, что не требуется?
Аноним 08/12/22 Чтв 14:23:44 568933 280
>>568929
Да. Из серии. Как ты собрался махать интерференцией волн, не зная что такое волна и что такое интерференция, а также что такое когерентность, и какие волны бывают?
Квантмех это не физика рыбникова. Он не вводит понятие православного атома. Это про то же, просто расширение понимания. В жёстких рамках, в той же системе определений и т.п. Это как не знаю. Сначала освоить интегралы и производные. Потом решать матрицы. И все это без арифметики, где учатся оперировать цифрами. И алгебры, где учатся оперировать еще и буквами.

Ты случаем не ойтишник?
Аноним 08/12/22 Чтв 16:35:43 568935 281
>>568932
> Так я и спрашиваю. Требуется ли она для понимания или нет, используется ли. Зачем учить просто так то, что не требуется?
Нет однозначного ответа. Можешь найти вузовский учебник, который почти с нуля будет объяснять, разве что математика там будет сразу вузовская. Другое дело, что имея школьные знания, банально проще и быстрее все пойдёт.
Аноним 08/12/22 Чтв 19:16:46 568937 282
>>568933
А и из школьной программы требуются вообще все знания или какие-то конкретные курсы? Атомы там, всё такое.

>Ты случаем не ойтишник?
Да, около того. Как угадал?
Аноним 08/12/22 Чтв 19:18:00 568938 283
>>568935
>Можешь найти вузовский учебник, который почти с нуля будет объяснять
А основные, Ландау/Лифшиц, или какие в основном используют в ВУЗах?
Аноним 08/12/22 Чтв 19:32:57 568939 284
image.png 12Кб, 878x342
878x342
Откуда это уравнение? Может это записанная по определению работа для одномерного пространства с силой, направленной противоположно нулю потенциальной энергии?
Аноним 08/12/22 Чтв 19:35:50 568940 285
>>568937
Пройди курс олимпиадной физики от Пенкина. Это вопрос пары месяцев, а потом будешь готов к всей вузовской сполна. Там на самом деле никакая не ужасная задротская олимпиадная физика, как у каких-нибудь математиков, а обычная, нормальная, какая и должна преподаваться в школах. Объясняется с нуля. Вся необходимая для вуза математика подхватывается там же.
Аноним 08/12/22 Чтв 22:52:33 568943 286
>>568937
Какой ты учебник собрался читать дурик? Может ты там картинки собрался увидеть ещё.
Учебник квантовой механики это одна сплошная математика практически без слов и текста вообще.
Соответственно должен в идеале знать вузовский выш мат и мат анализ.
А для этого очевидно должен в идеале знать всю школьную математику.
Если это знаешь то знание какие-то про атомы тебе не нужны, т.к. это всё равно математическая абстракция и ты поймёшь о чём речь просто глядя на формулы мат модели даже не зная слово из букв "атом"
Аноним 09/12/22 Птн 06:12:06 568948 287
>>568937
Школьную программу хотя бы экстерном пройти. Всю. Если до этого с физикой не соприкасался. Это, надо полагать, не составит труда. Раз за квантмех браться собрался. Что до айтишника, так это очень просто. Фраза:
>Зачем учить лишнее
Подход успешного вкатуна, которому не то чтобы и очень хочется, знать то, что он знает, учил и/или будет учить. Такой подход, пожалуй, свойственен для ойти более чем. Чтобы не устраивать срач, оговорюсь, что не даю оценку этому подходу. И не склоняюсь к тому, что это хорошо. Или плохо.
Аноним 11/12/22 Вск 23:16:43 569025 288
Откуда учоные в говне мочоные собираются доставать тонны трития для термояда? С дейтерием понятно, а тритий?
Аноним 11/12/22 Вск 23:18:34 569026 289
Что в 2022/2023 с солнечными панелями? Есть какие изобретения/прорывы?
Или так и сидят население/бизнес на кремнии а космос на галлий-индий-германии?
Аноним 11/12/22 Вск 23:45:38 569033 290
>>569025
Молодец, ты понял что это маняхуйня
Аноним 12/12/22 Пнд 01:32:11 569043 291
>>569025
>тонны трития для термояда
Из тритиевых брелков. Очевидно же.
Аноним 12/12/22 Пнд 03:00:35 569047 292
>>569043
Они реально тритиевые?
Это ж дорого пздц, а брелки дешовые
И распадается слишком быстро, явно что-то другое
Аноним 12/12/22 Пнд 04:22:58 569048 293
>>569047
>Они реально тритиевые?
Его там микрограммы. Еще и бодяжат криптоном его знатно.
>распадается слишком быстро,
Ну перестань. 12.5 лет Т1/2
Аноним 12/12/22 Пнд 07:52:02 569049 294
Почему медь такого красивого красного цвет?
Аноним 12/12/22 Пнд 11:00:17 569057 295
>>569049
У меди помимо электронов проводимости, которые делокализованы по всему куску, у каждого ядра есть особые загоны, куда при определенных условиях электроны проводимости могут залететь и остаться или наоборот. Собственно кванты света короче пороговой длиной волны заставляют электроны проводимости перейти в загон. Внешне это проявляется, что медь начинает сильно поглощать свет зеленый и синий свет, а красный наоборот хорошо отражает.
Аноним 12/12/22 Пнд 14:52:17 569061 296
Объясните фильм Довод. Как происходит реверс времени научно? Допустим, я вышел обратимым из камеры, меня в нормальном мире увидят только в конечной точке моего пути, то есть, например смерти. А значит мой труп оживёт, начнёт двигаться обратно к камере, поскольку я прошёл весь свой путь. Но ведь окружение в него может вмешаться снова? Например, забрать труп в морг. Как это работает вообще?
Аноним 12/12/22 Пнд 15:42:25 569068 297
16092369177360.jpg 151Кб, 1280x911
1280x911
>>569061
Как происходит реверс времени научно?
Аноним 12/12/22 Пнд 15:47:55 569070 298
»569061
Я думаю наука такое не описывает. Если хочешь поиграться со временем то есть СТО где нет сложных вычислений (максимум квадратный корень) и при этом это всё реально в нашем мире. Мозг сломает конкретно.

Я не понимаю: что не так с этими векторными произведениями? Они должны быть противоположно направлены, так как если нет, то выходит, что циклическая частота в квадрате отрицательна.
Аноним 12/12/22 Пнд 15:49:43 569072 299
image.png 407Кб, 2371x1503
2371x1503
>>569061 Я думаю наука такое не описывает. Если хочешь поиграться со временем то есть СТО где нет сложных вычислений (максимум квадратный корень) и при этом это всё реально в нашем мире. Мозг сломает конкретно.

Я не понимаю: что не так с этими векторными произведениями? Они должны быть противоположно направлены, так как если нет, то выходит, что циклическая частота в квадрате отрицательна.
Аноним 12/12/22 Пнд 15:55:09 569073 300
Аноним 12/12/22 Пнд 16:45:53 569075 301
>>569061
> Объясните фильм Довод. Как происходит реверс времени научно?
Там ненаучная хрень.
Аноним 12/12/22 Пнд 17:29:39 569077 302
изображение.png 696Кб, 690x679
690x679
>>567887 (OP)
Поясните за БЕЗРАЗМЕРНОСТЬ.

1. Это просто термин и размер все-таки есть, просто он не учитывается? Про электрон постоянно слышу, что он безразмерен.
2. Наткнулся на интервью Семихатова, он говорит, что у электрона нет боков, нет низа и вообще сторон. Но подождите, он же существует в ПРОСТРАНСТВЕ, даже если это некое подобие облака, и то мы можем пусть и приближенно, но сказать, вот это низ облака, а это его бок, разве нет?
3. Или это нихуя не облако? Или мы банально не знаем, что это?
4. Еще несколько раз видел изображение электрона в виде скопления точек и в какой-то статье тоже что-такое было. Это же не так, это некая единая, неделимая субстанция, а не скопление? Или точки это просто рассчитанные положения, где может находиться облако электрона?

Бонусный вопрос про энергию. Блять, что это такое вообще? Я понял, что любые взаимодействия это обмен частицами, тогда, что из себя представляет энергия? Может это типа материала, из которого состоят частицы?
Аноним 12/12/22 Пнд 17:31:47 569078 303
>>569077
>Или мы банально не знаем, что это?
Дополню вопрос номер 4 - если не знаем, то хули тогда пиздеть, что это облако? И тогда возвращаемся к изначальному вопросу - почему облака не может быть сторон, покуда оно существует в пространстве, если существует, конечно, если нет, то где оно тогда блять??
Аноним 12/12/22 Пнд 18:12:05 569079 304
>>569061
Да не особо он научный. Например, езда на необратимом автомобиле обращенного водителя вообще даже по логике фильма не понятно как работает. С чего вдруг обычное авто едет задом наперёд обратно во времени? Бензин и кислород в нем какие? Тоже обычные?
Аноним 12/12/22 Пнд 18:13:44 569080 305
>>569077
Электрон не облако. Это его положение, при попытке поймать, похоже на облако.
Аноним 12/12/22 Пнд 18:37:19 569082 306
Вопрос к геологу/географу какназываются вот эти белые скалы на видео ? Там два плана с ними. Я понимаю что это скалы останцы но какие именно ? На втором плане видно что они не особо большие и образуют по сути коридоры.
https://youtu.be/33MOJvnt1qY?t=1803
Аноним 12/12/22 Пнд 18:44:05 569083 307
Помню в ВУЗовской методичке по физике проводился мысленный эксперимент: ракета летит в чёрную дыру, для наблюдателя в ракете его туда "засасывает" прям вот в реальном времени; а для наблюдателя с земли в телескоп будет видно что ракета никогда не пересечёт горизонт событий, т. е. она всё ближе и ближе будет к "(затрудняюсь вспомнить к чему)".
Но ведь тогда по идее кажду чёрную дыру должно быть хорошо видно по затору из всего что она засасала когда либо вокруг неё, не ?
Аноним 12/12/22 Пнд 19:04:11 569084 308
изображение.png 1669Кб, 1627x911
1627x911
изображение.png 333Кб, 900x507
900x507
>>569080
Тогда я в растерянности, прочитал вот эту статью https://habr.com/ru/post/414925/#note3 Вкратце ее суть в том, что пустоты в атоме почти нет, за исключением ядра атома все пространство заполнено электроном. В таком случае если электрон не точка и не облако, то что это по форме? Ну точно не точка, насколько я понял.
Пикрилы по форме вполне облако, пусть и сферическое.
Аноним 12/12/22 Пнд 19:04:21 569085 309
>>569077
У точки в математике есть сторона? А низ? А верх?
Аноним 12/12/22 Пнд 19:05:23 569086 310
>>569084
Ты пытаешься реализмом, в квантовую механику лезть. Зачем? Без знаний квантов, ты это не поймешь, потому что это надо на уравнениях пояснять.
Аноним 12/12/22 Пнд 19:21:50 569087 311
>>569086
Уравнения это абстрактная модель. Но частицы существуют не в каком-то отдельном абстрактном измерении, а в той же реальности, что и макромир. Или все же нет? Вроде в теории суперструн была идея, что существует несколько суб-измерений, в которых частицы могут взаимодействуют. Значит они обладают формой, и пусть мы пока не способны наблюдать ее напрямую, но все же благодаря всем опытам в ускорителях заряженных частиц можем какие-то выводы на этот счет сделать. Опять же пикрилы >>569084 не какая-то модель или иллюстрация и на них мы видим объект, имеющий форму.
Аноним 12/12/22 Пнд 19:22:53 569088 312
>>569087
>Значит они обладают формой
Или на таком уровне реальности форма невозможна?
Аноним 12/12/22 Пнд 20:15:09 569092 313
>>569087
Ты же в курсе, что "форма" понятие из математики?
Аноним 12/12/22 Пнд 20:34:42 569093 314
Аноним 12/12/22 Пнд 20:42:32 569094 315
>>569079
>Бензин и кислород в нем какие? Тоже обычные?
Получается в том мире углекислый газ из воздуха набивался бы в любые замкнутые места и там взрывался бы разлогаясь на кислород и углерод, лол
Ведь никакой логики чтобы он из воздуха набивался именно в камеру сгорания двигателя через выхлопную трубу и там разлогался-нет
Аноним 12/12/22 Пнд 20:49:04 569095 316
>>569073
Завтра тебе сломают ноги если ты до 12 часов не достанешь надпочечник существа на кремниевой биологии.
Как ты собираешься это сделать?
Аноним 12/12/22 Пнд 22:48:24 569101 317
>>569087
Такие штуки как цвет и температура не существуют на микроуровне. Может и форма тоже не существует.
Аноним 13/12/22 Втр 00:33:17 569105 318
>>569101
Форма будет невозможна только в случае если там какое-то из измерений отключается/не работает, по-моему такое маловероятно. Хотя, конечно, наверняка сказать нельзя, форма зависит от наличия пространства, которого не было до большого взрыва, может на микроуровне пространство перестает работать или работает иначе как-то.
Аноним 13/12/22 Втр 02:44:50 569107 319
Для перехода в другое агрегатное состояние нужно затратить большое количество энергии, так? Можно как-то снять эту энергию при переходе из жидкой воды в лед?
Аноним 13/12/22 Втр 02:52:08 569108 320
>>569107
Определение "снять" можно?
Аноним 13/12/22 Втр 03:21:13 569109 321
Аноним 13/12/22 Втр 04:37:13 569111 322
Всегда ржу когда один и тот же Нобелевский лауреат чуть ли не в одном предложении использует фразу "Вселенная бесконечна" и через мгновение задвигает про теорию относительности.

Как людям на серьезной ебучке заявляющим прямо противоречащие друг другу вещи дают нобелевки вместо билета в дурку?

И вообще, мне одному кажется что наука свернула куда-то не туда?
Столько признанных научным миром заявлений опровергаемых простейшими и не подлежащими сомнению теориями и аксиомами что страшно как-то.

Короче! Красное это черное! Где моя ссаная нобелека????
Аноним 13/12/22 Втр 09:41:01 569115 323
>>569083
>хорошо видно по затору из всего что она засасала когда либо вокруг неё, не ?
Во-первых, весь этот затор должен светиться как звезда, чтоб его вообще можно было увидеть с большого расстояния. Во-вторых, изображение затора будет постепенно краснеть, а потом и вовсе исчезнет, так что по затору палить не выйдет. Я это из науччпоков с ютуба узнал, не бейте если неправда.
Аноним 13/12/22 Втр 09:55:27 569116 324
>>569105
Ещё формы нет, когда нет чётких границ.
Аноним 13/12/22 Втр 10:28:21 569119 325
>>569111
>не подлежащими сомнению
В этом твоя проблема. Точне, у тебя-то проблем нет: ну дурак и невежда, подумаешь. Проблема появляется, когда ты своими не подлежащими сомнению "теориями" пытяешься ссать в уши другим. Да и то проблема не велика. Никто не заставляет слушать даже самого тупого и наглого двачера - можно просто на хуй послать. Он же двачер: он же только на дваче срет, а за его пределами максимум кассиру пятерочки свою "теорию" изложит.
Аноним 13/12/22 Втр 15:41:29 569127 326
изображение.png 714Кб, 600x748
600x748
>>569077
Бамп вопросам!

>>569092
>>569085
Да поебать мне на математику. Ну т.е. безразмерность в отношении электрона это все-таки термин, а в реальности электрон обладает размером? А если размером обладает, то и форма должна быть.
И да, понятие "форма" появилось задолго до появления математики.


>>569116
У облака нет четких границ, но форма все же вполне различима.
Аноним 13/12/22 Втр 16:26:43 569132 327
>>569119
То есть движение на скорости выше скорости света возможно? Верно?
Аноним 13/12/22 Втр 17:57:08 569133 328
>>569111
Моська, приведи хоть один пример, когда учёный сказал, что вселенная бесконечна.
Аноним 13/12/22 Втр 18:03:00 569134 329
>>569133
А где её конец? Учёные то и дело постоянно пиздят, что даже конечные нейтроны нельзя определить в размере, а в квантовой физике вообще понятия размер нет. А здесь целая вселенная, которая ещё и РАСШИРЯЕТСЯ.
Аноним 13/12/22 Втр 21:40:39 569137 330
>>569109
Ты хочешь её на полезную работу пустить? Или просто в виде тепла?
Аноним 13/12/22 Втр 21:41:47 569138 331
>>569127
>а в реальности электрон обладает размером?
Никто не знает
Аноним 13/12/22 Втр 21:50:13 569139 332
>>569138
Есть граница после которой электрона становится мало и очень мало. Есть ли граница после которой электрона вообще нет неизвестно.
Аноним 13/12/22 Втр 22:04:23 569141 333
>>569134
Она может быть конечной и не иметь конца. Как поверхность шара.
Аноним 13/12/22 Втр 22:22:31 569146 334
>>569139
Граница электронного поля или электрического?
Аноним 13/12/22 Втр 22:42:37 569147 335
>>569048
И что вот прямо реально на реакторе нарабатывают чтобы в брилок засунуть?
Хотя учитывая что для бимб нужны граммы, если там микрограммы, то можно делать миллион брелков по 1$ с выгодой, ок
Аноним 13/12/22 Втр 23:25:57 569149 336
>>569141
У шара есть конец, просто мы бегаем по его краю, то есть на грани конца.
Аноним 14/12/22 Срд 00:03:40 569150 337
Аноним 14/12/22 Срд 00:27:07 569151 338
>>569150
Один прореагировавший атом на всю закладку это тоже плюс по энергии.
Аноним 14/12/22 Срд 01:20:33 569153 339
>>569107
Вот интересно конечно
Этот процес происходит с отбором энергии у вещества
При этом, оно при замерзании расширяется и может произвести работу

Это получается просто нелинейность в выделении запасённой энергии в веществе можно сказать?
Т.е. мы забираем энергию у тёплой воды, забираем, это то же самое что он выдаёт энергию, потом она вдруг выдаёт нам больше чем обычно в виде кроме тепловой ещё и движением, и потом опять выдает равномерно уже в виде льда?

А при обратном процессе происходит поглощение, поглощение, потом когда переход оно поглощает больше становясь водой
Оно не начинает во время перехода выдавать вместо поглощения?
Аноним 14/12/22 Срд 13:34:17 569170 340
image.png 463Кб, 1067x556
1067x556
image.png 101Кб, 805x611
805x611
>>567887 (OP)
Пояснительную бригаду географов плиз в тред.
Один буржуйский хуй прилепил в свою видюшку пик 1.
Это как понимать? Я на машине от таймыра по льдам могу до канады доехать?
Я полез искать карту арктических льдов
https://nsidc.org/data/soac/sea-ice-age
Что вообще из себя представляет арктический лед? Это корка сплошная без трещин а по периметру льдины кусками? Там вообще есть зона, где постоянный лед? Судя по карте полюс время от времени оттаивает, а вот кромка Гренландии и Канады всегда во льду.
Аноним 14/12/22 Срд 13:46:51 569171 341
Аноним 14/12/22 Срд 13:47:20 569172 342
Анончики-химики, помогите. Мне нужно убрать снежный покров на 10 квадратных метрах в лесу, чтобы оголилась земля. Причём очень аккуратно, потому что осенью я там проебал одну маленькую штучку и она лежит где-то там на земле, под снегом.

Приносил бутылки с горячей водой, но пока нёс 5л в полторашках, они остыли в рюкзаке и не особо это помогло. Был-бы рядом водоём, я бы вёдрами натаскал и тупо залил там всё, чтобы снег растаял, но водоёма рядом нет.

Может есть какие-нибудь химические экзотермические реакции, чтобы устроить их в ведре со снегом, которые растопили бы снег и я бы этой водой залил снег на земле, смывая его?

Пока из идей только купить таблеток сухого горючего, натопить в оцинкованном ведре снега и уже таким заливать. Не знаю, насколько это будет результативно.
Аноним 14/12/22 Срд 13:48:57 569173 343
>>569171
Скорее всего - нет. Так как в следствии потепления или механического воздействия лёд попросту может провалиться.
Аноним 14/12/22 Срд 13:54:14 569174 344
image.png 76Кб, 286x176
286x176
image.png 276Кб, 750x428
750x428
image.png 463Кб, 728x410
728x410
image.png 331Кб, 600x392
600x392
>>569170
>корка сплошная без трещин
Если морской лёд подвергался таянию хотя бы в течение одного года, он относится к старым льдам. Старые льды подразделяются на:

⋆ остаточный однолетний — не растаявший летом лёд, находящийся вновь в стадии замерзания,
⋆ двухлетний — просуществовавший более одного года (толщина достигает 2 м),
⋆ многолетний — старый лёд толщиной 3 м и более, переживший таяние не менее двух лет. Поверхность такого льда покрыта многочисленными неровностями, буграми, образовавшимися в результате неоднократного таяния. Нижняя поверхность многолетних льдов также отличается большой неровностью и разнообразием формы.

Исследование морского льда на Северном полюсе
Толщина многолетних льдов в Северном Ледовитом океане в некоторых районах достигает 4 м.
Аноним 14/12/22 Срд 13:55:33 569175 345
>>569172
Солью засыпь. Ещё лучше - хлористым кальцием.
Аноним 14/12/22 Срд 16:07:46 569185 346
>>569153
Первый закон термодинамики тебе говорит, что и в одну сторону и в другую одинаково всё. Если у тебя объем уменьшается, то тебе в виде тепла добавляется еще работа от внешнего давления, и наоборот

Второй закон термодинамики, запрещает нам из фазового перехода (при температуре фазового перехода) получать полезную работу. Хочешь тепло? Да, бери. Хочешь телефон зарядить? Пошел нахуй
Аноним 14/12/22 Срд 17:45:49 569190 347
16471902917360.png 1219Кб, 810x1080
810x1080
>>567887 (OP)
Приведите в пример ситуации, когда защитной реакцией одной колонии бактерий становится замедление размножения? На примере чашки Петри. Я не по ни маю
Аноним 14/12/22 Срд 18:26:34 569192 348
>>569190
Если это аналогия примера с людьми то она дважды глупая
Во-первых это только для более разумных существ чем бактерии может иметь толк
Во-вторых, это не выживание разных видов бактерий а один вид.
Аноним 14/12/22 Срд 18:50:28 569194 349
>>569026
Мне надоело это читать, т.к. все инфополе забито сообщениями типа "Британские ученые разработали с/панели в N раз легче/эффективний - рашка соснет." Именно так - почти всегда это если не подается так (с политическим выводом того или иного свойства), то хотя бы обсуждение скатывается в порашу.
Аноним 14/12/22 Срд 18:56:38 569195 350
Аноним 14/12/22 Срд 19:50:15 569196 351
>>569195
>бить нейтронами по ископаемому литию
Ясно. Я думал это рофл такой и существует нормальный способ.
Аноним 14/12/22 Срд 20:07:36 569197 352
>>569196
>ископаемому
Ты в курсе что всё ископаемое?
Или как ты это себе представлял, берём тратим овердохуя энергии чтобы из ничего получить что-то а потом с его помощью получаем 1% от затраченной энергии?
Аноним 15/12/22 Чтв 13:13:00 569213 353
>>567887 (OP)
как звучит выстрел в разреженной атмосфере? Например, на Марсе?
Аноним 15/12/22 Чтв 14:06:38 569214 354
image.png 49Кб, 499x722
499x722
Эта хуйня лженаука? Как вообще египтологи придумали фонетику то блять? Не я понимаю, расшифровать письмена по значению можно хоть как-то, но блять, фонетика... Я прокрутил колесиком сабж, и такое ощущение что придумали "чтоб созвучно было". Я хуею, у них такие научные подходы? Чет читать даже расхотелось.
Аноним 15/12/22 Чтв 14:26:04 569215 355
>>569214
Теоретически можно восстановить если есть родственные языки. Коптский вроде язык потомок древнеегипетского.
Аноним 15/12/22 Чтв 15:35:48 569221 356
>>569213
Как будет выглядеть наземный ядерный взрыв на поверхности Марса? А если в полном вакууме, например, на вершине Олимпа или Луне?
Аноним 15/12/22 Чтв 15:58:32 569222 357
>>569221
Пыль разлетится далеко и осядет очень быстро. А в (почти) вакууме взрывы проводили, погугли видео высотных и космических испытаний.
Аноним 15/12/22 Чтв 17:10:41 569228 358
>>569175
Купил, анончик, два кг соли. Завтра -3 днём, пойду пробовать.
Аноним 15/12/22 Чтв 20:46:48 569236 359
>>569215
А как ты проследишь? У татар и киргизов, произношения слов например нихуево так отличаются, понять это можно только из универсальности кириллицы.
Аноним 15/12/22 Чтв 21:56:53 569243 360
>>569149
> У шара есть конец, просто мы бегаем по его краю, то есть на грани конца.
Окей, меняем шар на сферу.
Аноним 15/12/22 Чтв 22:29:20 569246 361
>>569222
>Пыль разлетится далеко и осядет очень быстро
А как это будет выглядеть? Как грибок?

>>569222
>и осядет очень быстро
Почему?

>>569222
> погугли видео высотных и космических испытаний.
В том-то и дело, что я спрашиваю про наземный взрыв
Аноним 16/12/22 Птн 07:36:10 569277 362
>>568915
Получается куколд-порно.
Аноним 16/12/22 Птн 07:47:01 569279 363
Можно ли медикаментозно понизить температуру тела так, чтобы его не было видно в тепловизионный прицел? При этом чтобы поциент оставался полностью вменяем и активен, чтобы мог бегать в бронежилете и с автоматом.

Если скорее нет, чем да, то объясните кратко, какие процессы останавливаются в организме при переохлаждении, и почему. Интересует именно на молекулярном уровне - какая биохимия дохнет, и какие могут быть пути этого избежать?
Аноним 16/12/22 Птн 11:06:42 569289 364
>>567887 (OP)
Возможно ли в теории загенокодить ДНК таким образом, чтобы женщины в одном виде были одной расы, а мужчины совсем другой и это, разумеется, продолжалось дальше из поколение в поколение?
Половой диморфизм ведь есть, так почему бы не иметь еще и такой вот диморфизм по Y-хромосоме?
Аноним 16/12/22 Птн 11:23:15 569290 365
>>568894
И в итоге всё упирается в то, что математика есть, а понимания - нет.
Аноним 16/12/22 Птн 12:19:41 569295 366
>>569236
>Но тем не менее установлено что и татарский, и киргизский. и якутский, и турецкий относятся к одной языковой семье. Сравниваются схожести, например по русски будет Лес а по украински Лic, а в старой орфографии писалось через ять - то есть можно предположить что изначально там был звук средний между Е и И. И так далее в таком же духе.
>>569246
>А как это будет выглядеть? Как грибок?
>Почему?
Разряженная атмосфера меньше тормозит частицы пыли. На Марсе гриб еще возникнет но опадет гораздо быстрее чем на Земле. А на Луне пыль осядет так же быстро как и взлетела (кроме тех частиц которым хватит скорости уйти на орбиту).
>>569279
При снижении температуры на пару градусов с автоматом и бронежилетом ты уже не побегаешь, тебя захочется лечь в окоп и отдохнуть, при температуре ниже 20 градусов у окопчика можно уже будет воткнуть винтовку и повесить на нее каску. Индейские цветочки тут не помогут.
Аноним 16/12/22 Птн 12:30:34 569297 367
>>569290
Понимание есть, просто никто не хочет писать сюда посты на 60000 знаков, чтобы словами объяснять то, что с помощью формул помещается на три страницы.
Аноним 16/12/22 Птн 13:05:34 569301 368
>>569297
Да где оно есть? В миллиарде непроверяемых интерпретаций уровня фэнтези про миллиарды параллельных миров?
Аноним 16/12/22 Птн 13:19:44 569302 369
>>569301
Что ты вкладываешь в слово "понимание"? Можешь пример чего-то, где есть понимание?
Аноним 16/12/22 Птн 13:24:09 569303 370
>>569302
Если бы кванты работали как их иногда описывают (измерение = прибор вмешался = повлияли на частицу), то было бы понятно. Когда прибор вмешивается и целая новая вселенная появляется - не понятно. И да, я не дохуя философ (а нихуя не), чтоб тут тебе на те же 60000 знаков расписать значение слова понимание и само понимание.
Аноним 16/12/22 Птн 13:26:17 569304 371
>>569297
>Понимание есть
Ого! И за коллапс волновой пояснишь?

Напомню стандартные варианты ответов:
1) Коллапс? Какой еще коллапс? Где это я?
2) Нет никакого коллапса, врееееети!
3) Просто волновая функция существует только в голове наблюдателя это просто знания о системе, но при этом реальна и например описывается уравнением Шредингера - нет, это не шиза
4) Пук-сереньк, ты че охуел пидор тупой, ряяяяя.
Аноним 16/12/22 Птн 17:40:09 569312 372
>>569175
Засыпал пятью килограммами поваренной соли. Минус три. За джва часа ничего особо не растаяло, только места где была соль спеклись корочкой и немного продавились вглубь. Пойду завтра попробую всё-таки водичкой тёплой ещё полить.

Предвосхищая вопрос, лопаты у меня нет.
Аноним 16/12/22 Птн 18:45:56 569315 373
image 1105Кб, 1618x1412
1618x1412
>>569312
Бля. Кого ты слушаешь?
Мой совет.
Покупаешь много самой дешевой соли и садовую лейку. (Можно заехать в ДРСУ и попросить у них килограмм 5-7-10 дорожной соли, они ее один хуй кубами меряют)
На месте разводишь костер, топишь снег и делаешь рассол (Ну или тащишь уже приготовленный рассол) конц ~20%
Поливаешь этим рассолом из лейки твою поляну.
Аноним 16/12/22 Птн 20:44:22 569317 374
Почему аналогию кота шредера представляют как единичное событие, хотя принцип суперпозиции нагляднее всего проявляется в длительной эволюции системы? Допустим, у нас не кот, а немного беременная баба, пока мы не измеряем, она то ли родила, то ли нет, то ли умного то ли тупого, то ли гуманитария, то ли физика; ну и по остальным ветвям тоже развилки. И в конце типа спектр всех возможных исходов с разной вероятностью, которая отражает статистику в случае возможности повторить всё это много раз.
Аноним 16/12/22 Птн 20:56:05 569320 375
Аноним 16/12/22 Птн 21:48:01 569328 376
>>569295
Ну вот почему именно 36.6? Что за число Пи такое? Что при этой температуре происходит в организме такого, чего не может происходить при 20 градусах?
Аноним 16/12/22 Птн 22:36:00 569335 377
Аноним 17/12/22 Суб 04:09:28 569348 378
Вот у органов - глаза, печени и тд - есть чёткие границы, а зоны в мозге гораздо плавнее друг в друга переходят? Не говоря о том, что зрение и прочие чувства обрабатываются не только соотв участком коры, но много где ещё
Аноним 17/12/22 Суб 04:53:05 569350 379
Для зон мозга нет четкого забитого в ДНК плана. Мозг слишком большой и сложный, там на него места не хватит.
Нервы разрастаются, по мере необходимости, по всему организму, не только в мозгу.
Аноним 17/12/22 Суб 16:11:54 569365 380
>>569315
Не вышло анон. 3л солёной воды использовал. В общем, ладно, до весны оставлю это.
Аноним 18/12/22 Вск 11:11:03 569401 381
>>569365
Надо было сыпать хлорид кальция/магния + мраморную крошку, на крайний случай мел.
Аноним 18/12/22 Вск 11:13:25 569402 382
Сап.
Есть окно и уф-светодиоды.
Как отрезать/преобразовать уф-излучение в обычный спектр в каждом из случаев?
Аноним 18/12/22 Вск 11:17:44 569403 383
>>569402
Не понял, при чем тут окно?
Тебе нужно УФ в видимый свет преобразовать? Тогда тебе нужен обычный люминофор.
Аноним 18/12/22 Вск 13:15:59 569405 384
>>569304
>Ого! И за коллапс волновой пояснишь?
Зависит от определения, которое ты коллапсу дашь. Но ортодоксальная квантовая механика довольно просто это объясняет. Другое дело, что у нас нет достоверных знаний о том, работает ли квантовая механика на масштабах такого размера.
Аноним 18/12/22 Вск 13:29:07 569409 385
>>569403
Люминофор есть. Нужно, чтобыуф света не было видно после люминофора.
Аноним 18/12/22 Вск 19:42:48 569423 386
>>567887 (OP)
Сап.

Может, кто-то знает книгу (или другой источник), где объясняется зрение, его оптика с точки зрения "как форма глаза (и другие характеристики) влияет на получаемую в итоге картинку". И, возможно, там раскрывается такой вопрос, как "если один глаз это зрение без глубины, два это зрение трёхмерное, то может ли добавление ещё одного глаза добавить ещё одно воспринимаемое измерение?".

Благодарствую за внимание.
Аноним 18/12/22 Вск 20:33:51 569430 387
>>569423
>как форма глаза (и другие характеристики) влияет на получаемую в итоге картинку
При правильной форме глаза, фотоны попадают на правильную часть сетчатки. При неправильной, они до неё не долетают, или попадают не туда.
>два это зрение трёхмерное
Одним глазом невозможно отличить маленький объект от далекого и большой от близкого. С двумя, мозг может сравнить картинку и рассчитать расстояние. Третий уже ничего не добавляет.
Аноним 19/12/22 Пнд 10:03:20 569455 388
>>569430
>то может ли добавление ещё одного глаза добавить ещё одно воспринимаемое измерение?
С третьим глазом можно узреть суть.
>С двумя, мозг может сравнить картинку и рассчитать расстояние
На какой дистанции стереоэффект перестает работать? Ведь расстояние между зрачками сантиметров пять.
Аноним 19/12/22 Пнд 10:59:16 569458 389
>>569430
>При правильной форме глаза, фотоны попадают на правильную часть сетчатки
Имеется в виду, что с разной формой глаза может быть разная перспектива, дисторсия (как у линзы фишай). Или там предположительно решает софт, а не железо? Как, например, должна видеть мир пчела с её фасетками?
Аноним 19/12/22 Пнд 11:27:02 569460 390
>>569458
>решает софт, а не железо?
И то и другое как и везде. Зрачок может менять свою форму для фокусировки. Есть слепое пятно которое мозг старается игнорировать. Высокое разрешение имеет только очень маленький участок сетчатки, и чтобы все поле зрения было четким глаз постоянно движется.
Аноним 20/12/22 Втр 13:33:57 569526 391
Если у нас такое дикое потепление климата, почему не растёт влажность и Сахара не превращается в цветущую саванну?
Аноним 20/12/22 Втр 18:41:59 569567 392
>>569526
От потепления влага испаряется, а для конденсации ей надо охладиться. Поэтому Сахара и не превращается, влага испаряется, а обратно уже все, никак.
Аноним 20/12/22 Втр 18:58:11 569569 393
>>569567
тогда почему говорят, что влажность растёт при потеплении климата
Аноним 20/12/22 Втр 20:36:35 569570 394
>>569569
Впервые такое слышу, кто говорит? А так, там где еще прохладно, там и растет.
Аноним 20/12/22 Втр 20:40:47 569571 395
>>569569
Она растёт там, где вода есть, то есть в большинстве мест на Земле. Но не везде.
Аноним 21/12/22 Срд 09:27:53 569594 396
>>569213
>как звучит выстрел в разреженной атмосфере? Например, на Марсе?
бамп вопросу
Аноним 21/12/22 Срд 09:30:53 569596 397
Аноним 21/12/22 Срд 11:42:22 569604 398
image.png 34Кб, 491x290
491x290
>>569526
>>569569
Влажность растет за счет увеличения площади мирового океана. В целом планета стала зеленее, но в основном за счет экспансии лесов в полярные области.
По сахаре не нашел статистики по осадкам, но судя по графику в одной из статей площадь пустыни сокращается.
Аноним 22/12/22 Чтв 18:37:54 569679 399
>>567887 (OP)
Почему, когда к генератору постоянного тока, подключена нагрузка, его сложнее крутить?
Что за сила подтормаживает генератор, когда включена нагрузка?
Где возникает эта сила, и как она работает?
Аноним 22/12/22 Чтв 19:12:48 569682 400
>>569679
Чем сложнее пропихивать электроны тем легче крутить.
Аноним 23/12/22 Птн 00:15:16 569705 401
>>567887 (OP)
Можете пожалуйста простыми словами объяснить, что такое мультипликативность и аддитивность простыми словами.
И в чем разница между ними?
Аноним 23/12/22 Птн 00:43:00 569712 402
>>569705
Эм... куда проще то. Там определения очевидные просто

Ты бы еще спросил, что такое четность, очевидными словами
Аноним 23/12/22 Птн 00:57:51 569714 403
>>569712
Я чет не могу понять. Можешь ты объяснить...
Аноним 23/12/22 Птн 04:29:14 569721 404
Как такие реки, как Нил, Тигр, Евфрат текут тысячи километров через супержаркую суперсухую пустыню, десятки миллионов людей тратят воду на орошение, а эти реки не высыхают к хуям при этом? Почему вокруг Каспийского моря одни пустыни, да засушливые области? Это же море, там вокруг постоянно дожди должны идти. Какого хера тот же Крым засушливый, это же практически остров
Аноним 23/12/22 Птн 10:20:40 569729 405
Drawing.png 11Кб, 604x275
604x275
На музыкальном инструменте натянута струна например из 40 конских волос. Длина этой струны равна L см. Сила её натяжения равна например 5кг.

А вторая струна сделана из 20 конских волос. Длина и сила натяжения у неё такая же как у первой.

Первую струну скрутили 10 раз (как на рисунке). Сила её натяжения от этого увеличилась до 6кг.

Сколько примерно раз нужно скрутить вторую струну чтобы сила её натяжения тоже стала равна 6кг?

Не понимаю как это рассчитать или как загуглить хотя бы что-то что направит к решению.
Аноним 23/12/22 Птн 10:24:48 569730 406
maxg480c12r2x3p[...].jpeg 57Кб, 888x437
888x437
Объясните тупому на пальцах, почему прилив происходит и с обратной стороны земли, а не только с той, которая смотрит на луну? В гугле был, нихрена не понял.
Аноним 23/12/22 Птн 11:05:39 569731 407
>>569730
Так с той стороны же солнце тянет.
это сложная проблема
Аноним 23/12/22 Птн 11:09:25 569732 408
>>569721
>Почему вокруг Каспийского моря одни пустыни, да засушливые области? Это же море, там вокруг постоянно дожди должны идти. Какого хера тот же Крым засушливый, это же практически остров
За грехи предков.
Аноним 23/12/22 Птн 11:44:35 569736 409
Drawing2.png 81Кб, 1110x668
1110x668
>>569729
Дополню свой пост.

Представил насколько примерно растянется конский волос если развернуть поверхность струны (на рисунке волос обозначен синим) (дико извиняюсь за корявость).

Ещё условие задачи лучше изменить так, чтобы у второй струны было 10 волос, потому что если площадь сечения струны уменьшится в 4 раза (с 40 до 10 волос), то её диаметр, и длина окружности сечения уменьшатся в 2 раза. Так легче считать.

Что делать с этой информацией не знаю, и вообще походу не в ту степь лезу. Тут наверное проще всё намного? Помогите, а то мозги кипят.
Аноним 23/12/22 Птн 12:20:03 569741 410
Верно ли, что у животных страха смерти не существует просто потому, что они об этой смерти не имеют представления (да и человек весьма обстрактно её воспринимает, не как состояние себя)? Есть страх внешней угрозы (боли и удушья, прежде всего), который проявляется в увеличении тревоги, что чревато ещё большим, экспоненциальным её увеличением,- поэтому нужно любой ценой её уменьшить; может, тут ещё решают сопутствующие подъём давления и чувство нехватки воздуха. Ну короче, моя позиция, что ничего более сущностного, чем это нет, нет никакого отдельного страха смерти.
Аноним 23/12/22 Птн 12:33:35 569742 411
>>569721
Нил берет начало в той части Африки где постоянно идут дожди, Тигр и Евфрат стекают с гор где более влажный климат.
Климат у моря зависит от морских и воздушных течений. Климат в Крыму засушливый, а в расположенном на тех же широтах Приморье - влажный (а зимой еще и дубак). Самая сухая в мире пустыня - Атакама расположена у моря, поскольку из-за холодного течения в воздухе почти нет влаги.
Аноним 23/12/22 Птн 12:44:12 569743 412
Summer.Camp.Isl[...].jpg 137Кб, 772x434
772x434
>>569741
Может и есть, никто ведь у животных не ходил и не спрашивал.
Аноним 23/12/22 Птн 12:48:40 569745 413
>>569730
Да легко. Центр масс системы Земля - Луна находится не в центре Земли, а чуть в стороне. Поэтому, када эта система крутится, то сзади Земли получается центробежная сила, которая чуть уменьшает гравитацию и получается горб. Посчитай это дело и проверь, если интересно, а то примерно представить можно, но нужно и проверить, а то вдруг моченые наебали.
Аноним 23/12/22 Птн 12:53:37 569746 414
>>569741
Пиздеж. Все они понимают. Конечно не так отчетливо как мы, но понимают. Но это еще от животного зависит, от его осознанности. Вот насекомые - биороботы ебаные и действуют так как ты говоришь. Млекопитающие - понимают.
Аноним 23/12/22 Птн 12:55:34 569747 415
>>569741
> Ну короче, моя позиция, что ничего более сущностного, чем это нет, нет никакого отдельного страха смерти.
Сам страх заложен инстинктивно, что в людей, что в животных. Но у животных этот страх вызывает реальная угроза, а человек за счёт разума, рефлексируя, представляя свою смертность может сам себе вызвать этот страх.
Аноним 23/12/22 Птн 12:58:33 569748 416
>>569730
Воще настоящие береговые приливы в морях и океанах происходят не от Луны. Влияние Луны очень мало. Да и земля тоже вспучивается вместе с водой, это еще уменьшает заметность лунного прилива.
Аноним 23/12/22 Птн 13:07:53 569749 417
>>569729
Это из сопромата задачка? Прост не знаю на сколько ее принято упрощать. Если ее решать точно, то там трудно будет, всякие интегралы вылезут. Даже задумываться не охота.
Аноним 23/12/22 Птн 13:39:29 569750 418
>>569745
Читал подобную статью, скрин из неё, только про гравитацию там не слова. Почему центробежная сила проявляет себя только на противоположной стороне, когда она таким макаром должна быть на протяжении всей оси? В другой статье наоборот говорят, что с противоположной стороны образуется впадина, как ближайшая к земле кора вспучивается, так там впадывается и это место заполняется водой и поэтому там прилив, только схерали вода заполняет не просто освободившееся пространство, а натекает выше нормы?
>>569748
А от чего?
Аноним 23/12/22 Птн 13:40:22 569751 419
maxg480c12r2x3p[...].jpeg 70Кб, 888x646
888x646
>>569750
Отвалилось.
Забыл прилепить
Аноним 23/12/22 Птн 14:14:59 569752 420
изображение.png 78Кб, 863x566
863x566
>>569730
>>569750
Посчитал за тебя, не блогодори. Как видим ускорение сбоку Земли чуть выше чем сзади и спереди, значит вода и прочая магма будет стремиться туда и будут небольшие горбы. Причем ускорение спереди Земли чуточку меньше чем сзади, значит спереди горб будет на гулькин писюнчик больше.
Аноним 23/12/22 Птн 14:24:23 569753 421
>>569750
>А от чего?
Прецессия круговых морских течений от вращения Земли.
Аноним 23/12/22 Птн 14:25:57 569754 422
>>569752
Спасибо тебе больше. Только один вопрос.
>Причем ускорение спереди Земли чуточку меньше чем сзади, значит спереди горб будет на гулькин писюнчик больше.
-
>Как видим ускорение сбоку Земли чуть выше чем сзади и спереди, значит вода и прочая магма будет стремиться туда и будут небольшие горбы.
Может тогда наоборот оттуда, раз чем меньше ускорение, тем больше горб?
Аноним 23/12/22 Птн 14:35:03 569755 423
>>569754
Да, оттуда где больше ускорение туда где меньше. Земные молекулы друг на друга давят под этим ускорением и стремятся стать так как чтобы скомпенсировать все силы.
Аноним 23/12/22 Птн 14:46:47 569756 424
EXL110BT.jpg 35Кб, 482x314
482x314
>>569749
Не, у меня похожая проблема в реальне возникла, а задачку эту я сильно упростил чтобы всем понятно было. Сейчас попробую объяснить в чём конкретно моя проблема.

Если мы возьмём цельные струны из металла, то для того чтобы при их равной длине и равном натяжении был шаг в одну ноту - нужно чтобы диаметр каждой последущей струны был больше предыдущей в 2^(1/12) раз.

На муз. инструменте под названием "тальхарпа" нужен шаг в несколько нот. Если взять конские волосы/лески, то можно легко подсчитать сколько волос/лесок должно быть в каждой струне чтобы сила их натяжения была равной, но проблема в том что их необходимо скручивать, и вот тут-то вся загвоздка. Чем тоньше струна - тем больше скручиваний необходимо сделать, а сколько точно - неясно.

В старину толщину струн и число скручиваний никогда не рассчитывали, и в наше время тоже не рассчитывают, а подбирают опытным путём. Сила натяжения каждой струны при этом очевидно получается разной.

На гитарах и других муз. инструментах тоже с этим сильно не заморачиваются, но правда изготавливают и специальные струны с максимально приближенной друг к другу силой натяжения струн (на картинке).

Мне чисто из спортивного интереса захотелось выяснить, сколько скручиваний нужно делать на струнах из волос/лески, которые уже подобраны по толщине для равного натяжения, но пока ещё не скручены, хотя бы приблизительно. Я думал что эти скручивания легко рассчитать, и не надо учитывать материал струн, но похоже что это не так?

Извиняюсь за много букв.
Аноним 23/12/22 Птн 15:00:16 569757 425
>>569756
третий абзац, последнее предложение:
> Чем тоньше струна - тем больше скручиваний необходимо сделать
... для сохранения одинакового натяжения

дополнил чтобы было понятнее
Аноним 23/12/22 Птн 15:01:49 569758 426
>>569747

>>569746

Т.е. они конкретно боятся представления, что их функции организма могут прекратиться, сознание отключится, гнить начнут, на том свете бохнакажет? Из чего состоит этот страх смерти-то? У меня когда па были - именно сдохнуть несильно боялся, но боялся думать об этом, чтоб тревога не усилилась. Боялся лавинообразного усиления паники, что сойду с ума, задохнусь. Ещё никакие мантры не успокаивали, казалось, это не пройдёт, и даже после смерти легче не будет. Скорее, это боязнь некотнрорлируемой хаотической динамики, которая порвёт рассудок и останется только вопить в океане агонии. Т.е. в обычной активности человек и животное стремится устранить напряжение и уже этим намерением, привычной или рассудочной деятельностью, сосредоточением на цели состояние стабилизируется, непосредственный раздражитель чуть вымещается из сознания, контролируется и психикой, и гормоналкой. Совсем не то в тревоге и панике, когда в них полный разлад, и именно чувство потери контроля, что организм вразнос идёт мучительней всего и больше всего на "страх смерти" похоже.
А то какая-то метафизика представлять страх смерть как какуюто неделимую первосущность.
Аноним 23/12/22 Птн 15:02:00 569759 427
Я не понимаю, что значит эта точка, в которой собирается свет от предмета, прошедший через линзу? А в других не собирается, его не видно? Или типо не полная информация доходит от предмета?
Далее, когда мы имеем мнимое изображение, говорится что мы "видим предмет в этом месте". Я не понимаю этого. Поясните прошу. Как здесь правильней рассуждать?
Аноним 23/12/22 Птн 15:08:58 569760 428
>>569759
1 Просто получается уменьшенное изображение этого предмета, уменьшенное до размеров точки.
2 Мы видим там где его в реальности нет.
Аноним 23/12/22 Птн 16:36:02 569762 429
изображение.png 62Кб, 872x442
872x442
>>569752
Немного не правильно посчитал. Там где выделено исправил плюс на минус. Но общая суть от этого не поменялась. На масипусечку ускорения отличаются. Общий вывод что Луна очень слабо влияет на приливы, ведь отличия ускорений только в седьмом знаке.
Аноним 23/12/22 Птн 19:14:15 569772 430
Сколько в среднем в день человек производит писонья-каканья-пота-отмерших клеток вместе с мыльной водой, плюс воды от стирки белья? Это, видимо, всё через канализацию всё уходит в реку или океан? Там же органика в большинстве своём, правильно? Какие виды микроорганизмов, питающиеся всем этим начали процветать благодаря всем этим отходам? Или эти отходы как то ещё расщепляются, неорганическим образом или откладываются где то? Где и как? Из каких веществ всё это состоит?

И ещё - как разлагается пластик? Я слышал, что тысячи лет, это типо солнце нагревает и потихоньку его сжигает, выделяя углекислый газ и воду, правильно?

А что насчёт очисток от овощей, костей там всяких? Их кто разлагает?
Аноним 23/12/22 Птн 22:08:27 569779 431
>>569772
>А что насчёт очисток от овощей, костей там всяких? Их кто разлагает?
Бактерии. А пластик природные бактерии жрать не умеют, потому что не нужно было. Но сейчас наверняка такие искусственно выращивают, или пытаются выращивать, которые могут переваривать.
Аноним 23/12/22 Птн 23:06:58 569781 432
Объясните гуманитарию про спутниковую связь. Что мешает коннектиться с телефона к спутнику и получать 5G? Спутник далеко, облака, деревья, крыши и тп. мешают сигналу? Я так понял, что самое проблемное тут обеспечить связь в помещении? Типа бетонно-металлическая коробка глушит сигнал? В итоге в сычевальне сигнала со спутника ВООБЩЕ нет, или он такой плохой, что приемник не сможет из него выделить полезную информацию? Обладает ли человечество технологиями, чтобы это все же реализовать? Я не о коммерческой эксплуатации, а о принципиальной возможности
Аноним 23/12/22 Птн 23:14:49 569783 433
>>569781
Телефону надо не только получать, но и отправлять. Если тарелку к нему прикрутить, то особо ничего мешать на улице не будет.
Аноним 24/12/22 Суб 06:24:14 569788 434
>>569736
как я понимаю
1) за один оборот скручивания струна как бы становится короче на длину окружности, вокруг которой она оборачивается (плюс какой-нибудь коэффициент жёсткости/вязкости/хз, его не учитываю)
2) две толстые струны оборачиваются по одинаковой окружности диаметром в одну струну, то есть "становятся короче" на π*d
3) если струн пучок, то там они все оборачиваются по разным окружностям в зависимости от расстояния до центра пучка. Центральная жила не укорачивается, остальные — чем дальше, тем сильнее
имеет смысл?
Аноним 24/12/22 Суб 07:40:45 569792 435
>>569781
Телефон до спутника не добивает. Конечно, как сказал анон выше, можно прикрутить тарелку и сигнал с телефона будет долетать, но не эффективнее ли не каждому телефону давать тарелку, а вынести эти тарелки на специальные вышки и расставить их по городу? Вот до вышек уже телефон достает, а они, в свою очередь, достают до спутника. В заложенном диапазоне частот, поэтому для того, чтобы с ними работал г5, их надо модифицировать, а то и проще новые ставить.
Аноним 24/12/22 Суб 09:23:05 569795 436
redditsavecomhe[...].mp4 1546Кб, 574x720, 00:00:05
574x720
>>567887 (OP)
Я нипанимат, почему в сообщающихся сосудах, разный уровень воды
Аноним 24/12/22 Суб 11:17:58 569798 437
>>569742
смотри, у дельты Нила в древности было 8 рукавов, а сейчас всего 2. Это что, Нил пересыхает что ли так активно? Могут эти дожди у истока Нила когда-нибудь кончиться? Интересует перспектива не в миллионы лет, кончено же, а в ближайшие столетия или хотя бы десятилетия
Аноним 24/12/22 Суб 12:01:56 569799 438
>>569795
на шебмрил не вполне сообщающиеся сосуды.
Открытый сосуд все время находится под атмосферным давлением.
В закрытом сосуде при оттоке воды падает давление воздушного пузыря.
Таким образом, давление уравновешивается при разных уровнях воды в сосудах.
Аноним 24/12/22 Суб 12:03:44 569800 439
>>569798
>а сейчас всего 2
Возможно это связано с Асуанской плотиной. Ну и вобще русла рек могут довольно сильно меняться со временем.
>Могут эти дожди у истока Нила когда-нибудь кончиться?
Из-за глобального потепления климат меняется, где-то становится суше, а где-то наоборот влажнее. Как это проявляется у истоков Нила мне не известно.
Аноним 24/12/22 Суб 17:21:35 569816 440
Drawing3.png 48Кб, 970x678
970x678
>>569788
Возможно я неправильно понял, но если струну/нитку обернуть один раз без растягивания вдоль вокруг цилиндра, то расстояние на которое она станет короче будет зависеть от её длины.

Я даже взял нитку и проверил это на всякий случай на ножке от стула. Если взять короткую нитку, то она укоротится сильнее, а длинная нитка укоротится меньше.

Вот даже примерная формула для расчёта укорочения нитки без растягивания при одном обороте вокруг цилиндра (на картинке). Если в эту формулу подставить длину нитки L = 20см, а длину окружности цилиндра C = 10см, то укорочение будет примерно 2,68см. А при длине нитки L = 40см на том же цилиндре укорочение этой нитки будет уже примерно 1,27см.

А насчёт разности в натяжении волос в скрученной струне:
В реальности те волосы, которые находятся у поверхности струны и натянуты сильнее - они стремятся к середине и вытесняют менее натянутые волосы в сторону. В итоге натяжение волос более-менее выравнивается. Короче тут возможно всё очень сложно.
Аноним 24/12/22 Суб 18:13:27 569818 441
>>567887 (OP)
Что первично поля 4 взаимодействий физики или частица?
Если я правильно понял частиц без поля не бывает, поле без частиц бывает. Так?
Аноним 24/12/22 Суб 18:57:54 569820 442
>>569818
Энергия вселенной ноль, масса а значит и материя это разновидность энергии, а значит материи во вселенной нет, во вселенной ничего нет.
Аноним 24/12/22 Суб 19:00:41 569821 443
>>569818
Частицы сделаны из полей значит поле первичней.
Аноним 24/12/22 Суб 23:54:31 569837 444
>>569818
Поля. Частицы это только представление полей.
Аноним 25/12/22 Вск 04:55:44 569861 445
>>569837
А что тогда из себя представляет само поле? из какой сферической и плотноприжатой херни?
Аноним 25/12/22 Вск 07:43:23 569862 446
>>567887 (OP)
Анон, подскажи!
Чё это за боты доебались ко мне, с раннего детства, блядь?
Какие-то дегенератские, козлосучие, ссыкливые и уёбищные боты циничные...
Бегают, со своей непрактичной слежкой по высокоточной модели объективной реальности, лезут по модели этой, на опережение, и исподтишка цепляются ко мне,
навязывая в нервы всякое падло, и творят его, замыкаясь в падло это.
Что это за хуйня уёбищная? Какой-то ИИ, что-ли?
Я так полагаю, это какие-то жидо-фашистские боты, либо пидорашкинские.
С другой стороны, это походит на пидорашкинских говноботов.
Смотри, анон, я родился в православной цивилизации, и был крещён, после этого замкнулись на меня эти ссыкливые боты, и начали воротить дегенератскую хуйню всякую, пересерая мне всё к херам собачьим, исподтишка. Чё за говно? Это пидорашкинские боты штоле? Или жидовские? Может пендосранские? Хотя не, вряд-ли это пендосранские боты, потому что пендосранские боты не моделировали бы мне тухчарску резню, расстрел сомалийских пиратов, и всякие терракты 11-го сентября, а предотвратили бы их, владея инфой с модели. С другой стороны, я понимаю, что из-за непрактичности слежки этой бессмысленной, нихуя невозможно было бы предотвратить, потому что эти боты, фиксируя к продавливанию падло всякое - интерпретируют инфу с модели в качестве инструкций к неукоснительному и фанатичному исполнению, формируя это падло блядь. Таким образом, вполне возможно, что это был и пендосранские боты. Ну или пиздоглазые боты тоже, так как в китае есть суперкомпы.
Но это не точно.
Поясняй давай, что это за говноботы, и что за хуйня у них в бошки ихние безмозглые вставлена?
Действительно ли эти выблядочные нищие боты крысинные, необучаемы? И как нам быть с этим говном неисправымым? Они что позамыкаются нахуй друг на друга, просто потому что так принято У НИХ?
Аноним 25/12/22 Вск 08:58:31 569867 447
>>569862
А на русский перевести можно? Это паста что-ли?
Аноним 25/12/22 Вск 09:27:41 569868 448
>>569861
Никакой, колебания поля просто.
Аноним 25/12/22 Вск 12:43:24 569873 449
Kitten hugged l[...].mp4 6810Кб, 1280x720, 00:02:03
1280x720
>>567887 (OP)
Если кошак может быть живым и одновременно мертвым, то в чем проблема запилить такую интерпретацию квантовой механики, которая являлась бы суперпозицией всех интерпретаций? Почему считают, что интерпретации квантовой механики исключают друг дружку?
Аноним 25/12/22 Вск 13:05:15 569875 450
>>569873
>считают
Кто, блядь, так считает, кроме посетителей треда тупых вопросов? Значение слова "интерпретация" знаешь?
Аноним 25/12/22 Вск 14:03:11 569881 451
>>569799
Спасибо, умный человек
Аноним 25/12/22 Вск 17:48:05 569885 452
Да, это так.webm 185Кб, 640x360, 00:00:02
640x360
Аноним 25/12/22 Вск 19:02:40 569887 453
>>569861
В самом простом случае поле это функция от координат. В более общем случае это геометрический объект.
Аноним 25/12/22 Вск 19:53:44 569890 454
Почему Teleforce Николы Теслы невозможен? Зарядить кусочек металла размером с песчинку и ускорить его напряжением - вроде никаких подводных.

https://ru.wikipedia.org/wiki/Teleforce
>Записи Тесла указывают, что устройство основано на узком пучке небольших частиц вольфрама, которые ускоряются при помощи высокого напряжения (посредством устройства похожего на его усилительный передатчик).[2] Частицы, выпущенные из трубы электростатическим отталкиванием, способны лететь со скоростью в 48 раз превышающей скорость звука.
Аноним 25/12/22 Вск 20:25:01 569893 455
>>569887
Это математически. А физически что такое поле?
Аноним 25/12/22 Вск 20:27:17 569894 456
Аноним 25/12/22 Вск 20:40:38 569897 457
>>569890
Скорее всего там дохуя инженерных проблем типа ебовейшие поля, необходимость очень точных таймингов включения-выключения тока и то, что малейшие ошибки в расчётах или в исполнении приведут к распидорашиванию всей установки.
Аноним 25/12/22 Вск 20:43:19 569898 458
>>569890
В принципе возможен, боле того разные виды оружия направленой энергии (это устоявшийся термин) именно на ускорения различных, в т.ч макроскопических, заряженых частиц разрабабывали всю холодную войну, но:
1) Там много нюансов (например надо нейтрализовать заряд частиц после вылета).
2) Это хорошо для вакуума (для применения в атмосфере предпологалось "выживание" канала мощным лазером и только на относительно небольшие расстояния).
3) Если частицы вольфрама достаточно крупные это не принципиально отличается от любой электромагниьной пушки (только способ ускорения неудобный - электростатический).

https://ru.m.wikipedia.org/wiki/Пучковое_оружие
Аноним 25/12/22 Вск 20:52:49 569899 459
>>569890
>>569898

Ну и да, самое главное - можеш попробовать посчитать как быстро будут тормозит я снаряды летящив в атмосфересо
>скоростью в 48 раз превышающей скорость звука
в зависимости от размеров, формы и скорости снарядов.

Даже без формул понятно что выгодно делать не микрочастицы которые относително быстро затормозятся (а то и сгорят), а большой стреловидный снаряд - но его электростатическим методом не разогнать.
Аноним 25/12/22 Вск 20:56:31 569900 460
>>569898
> "выживание" канала
" "выжигание" канала
Аноним 25/12/22 Вск 21:12:30 569901 461
1958179.jpg 221Кб, 1199x795
1199x795
Приведите хотя бы один довод, почему теория тёмной материи более верна, чем MOND.
Аноним 25/12/22 Вск 23:15:00 569902 462
>>569901
Потому что монд звучит нереспектабельно, как мондовошка какая-то.
Аноним 25/12/22 Вск 23:28:38 569903 463
>>569901
Вроде нашли галактику, которая ведёт себя, как если бы в ней не было тёмной материи. Если бы MOND была верна, такой галактики не могло бы существовать.
Аноним 25/12/22 Вск 23:50:15 569905 464
>>569893
Т.е. когда ты изучал в школе как физические точки (мат.объект) двигаются по траекториям (мат.объект), подчиняясь законам Ньютона (дифференциальное уравнение), для тебя это было физически, ясно
Аноним 26/12/22 Пнд 03:51:13 569908 465
>>569867
Это не паста, и я конкретно у вас спрашиваю, что это за хуйня, блядь, дегенератская мне на голову вылезла исподтишка?
Что за боты ссыкливые? Хули они такие мерзопакостные блядь? Что это за ИИ ебанутый такой? Хули алгоритмика такая уёбищная? И как фиксить это говно, блядь? Или оно неисправимо, сукаблядь?
Аноним 26/12/22 Пнд 10:37:29 569918 466
>>569800
А до глобального потепления климат не менялся?
Аноним 26/12/22 Пнд 11:48:57 569919 467
Аноним 26/12/22 Пнд 11:54:18 569920 468
>>569905
Описание движения никаких проблем не вызывает, тк это всего лишь описание. Сами объекты при этом вполне себе физические и не вызывают вопросов, тк никто не заменял физический объект на математический.
Тут же пытаются заменить физический объект (частицу например) на математический объект - поле. Это вызывает вопросы.
Аноним 26/12/22 Пнд 12:05:07 569921 469
>>569918
Менялся. В средние века был например малый ледниковый период. В Сахаре процветал город Тимбукту, а на месте нынешних среднеазиатских пустынь декхане радостно выращивали хурму. В Московии все как всегда мерзли. А в Гренландии викинги нафиг вымерзли на радость эскимосам.
Аноним 26/12/22 Пнд 13:02:42 569925 470
>>569894
Алгебраическое поле к полям тензорным отношения не имеет (даже учитывая, что о категории расслоений можно думать как о категории конечно порождённых проективных модулей над кольцом функций). Если в математике не разбираешься, лучше сидеть тихо и анонов с толку не сбивать.

>>569919
Теория поля - это математическая модель. Невероятно точная и очень близкая к тому, что мы наблюдаем, но это модель. Твой вопрос - это "а как оно на самом деле?" в другой обёртке, потому что даже если ты интуитивно не понимаешь, что такое поле, определения частицы ты тоже не дашь. Физики под такими понятиями имеют ввиду совершенно конкретные вещи (например, неприводимые представления), которые с бытовой интуицией "маленького шарика" не имеют ничего общего уже больше столетия.
Аноним 26/12/22 Пнд 15:01:27 569932 471
>>569920
Математика точнее чем реальность. Наше восприятие реальности какую точность имеет? Ну вот твои представления о предметах вокруг? Они ограничены органами чувств. Математика же их описывает с точностью, в миллиарды-миллионов раз точнее, чем ты их чувствуешь. Поэтому она более реальна.
Двач-помогач, выручай дауна! Аноним 26/12/22 Пнд 15:14:00 569933 472
2022-12-2610-30.png 57Кб, 1186x391
1186x391
Notyou~1.jpg 70Кб, 562x841
562x841
Нужно подготовиться к зачёту по механике сплошных сред и тензорному исчислению за ночь. ОП - хуй и даун, сдавать будет злобному деду. Не знает нихуя, но объём вроде небольшой и в наличии хорошая кратковременная память.
Накидайте советов и хуёв за щёку
Аноним 26/12/22 Пнд 15:16:17 569934 473
>>569933
Иди в армию

Помогать тебе, это мешать СВО
Аноним 26/12/22 Пнд 15:25:35 569936 474
tumblr47c605a15[...].jpg 381Кб, 1080x1080
1080x1080
>>569934
У меня ожирение и шизофрения, няш.
Аноним 26/12/22 Пнд 15:28:53 569938 475
>>569934
>Помогать тебе, это мешать СВО

Сосать русня. Как там Херсон поживает?

Тестирую, как быстро появятся лахто- и ципсоботы
Аноним 26/12/22 Пнд 15:36:09 569939 476
>>569936
Значит будешь отвлекать противников на себя
Аноним 26/12/22 Пнд 15:57:40 569940 477
>>568575
Зумеры изобрели фотосинтез
Аноним 26/12/22 Пнд 20:36:31 569954 478
если зайти в химлабораторию и навернуть рандомную пробирку вещества, какая вероятность помереть? а если две разные пробирки - вероятность может быть ниже, потому что, может, вещества частично нейтрализуют друг друга?
Аноним 26/12/22 Пнд 21:06:09 569955 479
>>569954
50%, либо помрёшь, либо нет.
Аноним 27/12/22 Втр 09:11:13 569958 480
>>567887 (OP)
Кто-нибудь может скинуть мне DOI статьи где бы описывалось нормально в чем состоит последняя самая актуальная модель климатологов. От того что я нахожу в google scholar у меня глаза вытекают.
Аноним 27/12/22 Втр 12:26:31 569962 481
>>569958
>самая актуальная модель климатологов
Срочно налоги на пердёж ввести и раскулачить фермеров, а не то через 10 15 20 30 лет точно все помрём, мамой клянусь.
Аноним 27/12/22 Втр 13:11:20 569964 482
>>569861
Ну физическое поле это распределение какого либо параметра материи по пространству.

Вот есть простанство. Есть материя (эфир). Материя заполняет это пространство. Материя обладает каким либо параметром, например, температурой. Распределение температуры эфира по пространству есть гравитационное поле. Если в рандомном маленьком месте померить температуру эфира то это называется напряженность гравитационного поля в этом месте. Аналогично можно брать другие параметры материи и обзывать их распределение по пространству какими-либо полями.
Аноним 27/12/22 Втр 14:19:15 569966 483
image.png 101Кб, 300x300
300x300
Аноним 28/12/22 Срд 02:52:01 569975 484
Плиз, дайте офф ссылки на все варианты двухщелевых экспериментов.
Аноним 28/12/22 Срд 08:29:08 569979 485
Ну вот достроят ITER к 2030 году и что, правда учёные за 10-15 лет испытаний на нём узнают все нюансы термоядерного синтеза и после этого можно будет делать потребительские реакторы для вырабатывания электроэнергии?
Аноним 28/12/22 Срд 09:20:49 569980 486
>>569975
В вики недостаточно что ли?
Аноним 28/12/22 Срд 09:22:00 569981 487
Аноним 28/12/22 Срд 10:45:15 569985 488
Аноним 28/12/22 Срд 10:47:25 569986 489
7y.png 70Кб, 412x272
412x272
Когда хоть ещё одна задача тыщалетия будет решена? До 2050 справятся?
Аноним 28/12/22 Срд 12:41:31 569987 490
>>569986
Математики сидят в /math/.
Аноним 28/12/22 Срд 13:41:18 569990 491
>>569985
Ты опять забыл таблетки принять?
Аноним 28/12/22 Срд 14:13:19 569993 492
Аноним 28/12/22 Срд 15:10:47 569997 493
>>569955
это не вероятность, а количество исходов же((. а вероятность - это высчитанный приоритет исходов в отношении друг друга
Аноним 28/12/22 Срд 20:35:56 570011 494
Аноним 28/12/22 Срд 23:49:36 570014 495
>>569986
Какая разница что там математики себе наманяфантазировали? Настоящие проблемы тысечялетия это физические проблемы.
Аноним 29/12/22 Чтв 00:44:02 570016 496
Что за херня во всех статьях и картинках с таблицами когда в гугле вбиваешь "фундаментальные константы"?
Что бля они туда позапихивали...

Какие константы РЕАЛЬНО фундаментальные?
Аноним 29/12/22 Чтв 08:33:23 570021 497
>>570016
Если постоянную планку и скорость света считать за единицу, то прям фундаментными константы это величины силы взаимодействий. Для электромагнитного даже есть специальное название постоянная тонкой структуры. Ну еще есть матрицы смешивания фермионов.
Аноним 29/12/22 Чтв 11:15:44 570023 498
>>570016
До фундаментальных еще не докопались. То что ща есть это не константы, а переменные.
Аноним 29/12/22 Чтв 13:44:31 570026 499
>>567887 (OP)
Перед сном я как обычно загореля с тупизны комиксодаунов с их суперменами и с верунов в экстрасенсорику, затем немного задумался -- а каким образом вообще чел в одном лице может доминировать над многими. Телепатия...
A ведь если допустить, что телепатия существует и допустим твой мозг испускает какие-то особенные волны там, то в этом все равно не будет никакой пользы для тебя, правда? Ведь волны мозга все равно слишком слабые и не смогут ни на кого воздействовать.
Аноним 29/12/22 Чтв 14:37:35 570031 500
>>570026
>если допустить, что телепатия существует
Если допустить, то можно допустить все что угодно - телепатию, телекинез, абсолютное всемогуторство и наличие у меня тян.
Если же речь идет о суровой реальности, то электрические волны испускаемые мозгом улавливаются лишь чувствительными датчиками. А вот к этим датчикам уже вполне успешно приделывают всякие штуки, и можно мысленно управлять ими (например двигать курсор по экрану).
Аноним 29/12/22 Чтв 19:46:33 570034 501
>>570026
>волны мозга все равно слишком слабые
маленький камешка делать лавина
Аноним 30/12/22 Птн 10:11:28 570046 502
>>570031
>и наличие у меня тян
Не. Ну это сынок совсем ненаучная фантастика.
Аноним 30/12/22 Птн 10:16:07 570047 503
>>570031
>телепатию
Внутреннее ухо каждого человека представляет собой уникальной формы резонансную полость.
Я почти уверен, что современные нейроинтерфейсы можно настроить на распознавание образов и привязать к ним передатчик, который бы, модулируя волны, которые резонируют с твоим резонатором, мог передавать тебе чужие мыслеобразы так, что ты бы их воспринимал как речь в голове.
Но мне лет 10 назад было лень этим заниматься.
Лень и сейчас.
Аноним 30/12/22 Птн 12:12:50 570052 504
Сейчас готовил хавку и задумывался, какая прихватка будет хуже проводить тепло от горячей рукоятки кастрюли до руки - влажная или сухая? Потому что вода, насколько я знаю, очень теплоемкая, и по идее рукоятка заебется ее нагревать. Но с другой стороны влажная прихватка будет более плотной - а соответственно тепло будет продвигаться к руке быстрее.
Аноним 30/12/22 Птн 13:13:24 570053 505
>>570052
>вода, насколько я знаю, очень теплоемкая
Количество воды в прихватке очень небольшое и она быстро нагреется, с другой стороны вода испаряясь будет охлаждать прихватку. Короче нужно ставить полномасштабный эксперимент в духе разрушителей легенд (чтоб в конце обязательно взрыв)
Аноним 30/12/22 Птн 13:24:51 570054 506
>>570052
Эмпирически: влажная, мокрая прихватка – это пиздец.
Аноним 30/12/22 Птн 16:17:22 570066 507
Может ли пространство быть бесконечным "вглубь"? Т.е. если бы теоритически у нас появилась машина, способная уменьшать любой объект, то в таком случае объект эксперимента мы бы уменьшали бесконечно.
Аноним 30/12/22 Птн 17:01:38 570069 508
А где-то вообще можно почитать каким именно образом снимаются данные с детекторов пролетающих частиц в двухщелевом эксперименте?
Аноним 30/12/22 Птн 20:35:35 570076 509
>>567887 (OP)
А почему при отоплении сушится воздух? Ведь при повышении температуры повышается и относительная влажность, то есть и влажности должно быть больше
Аноним 30/12/22 Птн 22:38:33 570080 510
Мне кажется или на земле слишком дохуя кислорода?
На первый взгляд кажется что всё в оксидах. Т.е. куча разных элементов в связи с одним кислородом.
Песок весь оксид, водород весь в оксиде, железо всё в оксиде...
С другой стороны, ядро может чисто железное, да и хотя там такая температура, железное короче. Кремний может ещё где-то там в глубине есть не в виде оксидов...
В общем, смотреть надо состав планеты целиком, но в любом случае не слишком ли дохера кислорода?
Его что, при взрывах сверхновых образуется гораздо больше в пропорции к другим элементам?
Аноним 30/12/22 Птн 22:40:05 570081 511
>>570076
>повышается и относительная влажность
Или ты не так прочитал, или статью там где ты читал писал долбаёб
Аноним 30/12/22 Птн 22:42:29 570082 512
>>570066
Планковский размер, планковская плотность
Аноним 30/12/22 Птн 23:01:16 570084 513
15729776756270.png 98Кб, 1267x785
1267x785
Я без иронии считаю, что math is racist. И поэтому необходимо запретить её к преподаванию в учебных заведениях для евреев. А негров учить в отдельных спецшколах без матана.
Аноним 30/12/22 Птн 23:18:13 570085 514
>>570084
Интереснее другое. Когда у девушек есть возможность выбирать, то это расизм. Получается права женщин это расизм.
Аноним 30/12/22 Птн 23:55:34 570086 515
>>570081
А у меня в инструкции к увлажнителю воздуха есть табличка.
Температура воздуха и влажность %
20-21 = 65%
26-27 = 50%

Как это рассчитать?
Аноним 31/12/22 Суб 01:02:41 570087 516
>>570086
Влажность воздуха относительная величина, равная отношению давления паров воды к давлению насыщенного пара. Чем выше температура, тем выше давление насыщенного пара. Поэтому если у тебя при 20 градусах 65%, то при повышении температуры, такое же количество воды в воздухе будет давать меньшую влажность.
Аноним 31/12/22 Суб 01:03:52 570088 517
>>570080
Легкие элементы "всплыли" из недр планеты, как всплывает масло в смеси с водой.
Аноним 31/12/22 Суб 01:33:36 570089 518
>>570088
Т.е. кислорода много только на поверхности?
Аноним 31/12/22 Суб 01:40:16 570090 519
>>570086
Если нагреть один и тот же воздух, а это значит раз он один и тот же то и количество воды в нём то же, то относительная влажность упадёт, т.к. ёмкость воздуха тёплого больше чем холодного, он может вместить больше воды. Вот относительная влажность и упала, хотя количество воды осталось прежним.
А ты написал
>>570076
>повышении температуры повышается и относительная влажность
Это не сходится с тем что ты тут пишешь
>20-21 = 65%
>26-27 = 50%
Она ж уменьшилась, влажность. Относительная. А абсолютная осталась такая же. Вот если взять с относительной влажностью воздух, нагреть его так, чтобы относительная влажность не упала, тогда абсолютная влажность уже повысится.
Т.е. абсолютная влажность при нагреве увеличивается, если не изменилась относительная влажность. Но это откуда-то при нагревании должна извне поступать вода в воздух.

Рассчитать в калькуляторе в гугле
Аноним 31/12/22 Суб 21:16:45 570113 520
Тупой вопрос:
Поясните за религиозность и веру в высшие силы в научном сообществе среди представителей разных наук. Встречал инфу что почти все ученые это материалисты-сциентисты-атеисты (в основном от атеистов), а так же и опровержение этой инфы, мол процент верующих там нормальный, и больше зависит от окружения (страны и города в котором люди живут). Еще натыкался на инфу что многие известные ученые увлекались мистицизмом (Ньютон, Тесла и т.д.) и что это в целом не редкость. Так ли это? И как обстоят дела в этой области на 2022 год?
Аноним 01/01/23 Вск 06:22:22 570152 521
>>570113
>мол процент верующих там нормальный
Вряд ли. В любом случае процент верующих ученых будет меньше общего процента верующих в данном обществе.
>что многие известные ученые увлекались мистицизмом (Ньютон, Тесла и т.д.)
Во времена ньютона процент атеистов был исчезающе мал. Сам Ньютон был вполне себе религиозным человеком. Ну а во времена Теслы увлекаться всяким спиритизмом и прочей блаватщиной было очень модно.
Аноним 01/01/23 Вск 18:08:22 570184 522
>>570152
Ньютон свои занятия наукой считал сущей мелочью незаслуживающей внимания по сравнению со своими теологическими изысканиями. "вполне себе религиозным человеком" — это очень мягко сказано.
Аноним 01/01/23 Вск 20:39:08 570186 523
9bc6680e-6c87-4[...].jpg 28Кб, 640x360
640x360
stolp-plameni-s[...].jpg 595Кб, 1440x2560
1440x2560
Может ли человек значительно увеличить свою силу используя дыхание, как это показано в джоджо или клинке рассекающем демонов? То что увеличить ее незначительно возможно, это я знаю.
Аноним 01/01/23 Вск 20:40:21 570187 524
Химия Аноним 02/01/23 Пнд 12:14:26 570204 525
Почему человек пьянеет от алкоголя? Вот берём ситуацию, что человек берёт и выпивает чашку алкоголя. Алкоголь попадает в желудок, а в желудке желудочный сок, то есть кислота. Алкоголь - это спирт, в определённом смысле, щёлочь. Спирто-щелочные окончания -OH и кислотные H+ дают нейтрализацию и воду H2O. Откуда берётся опьянение?
Аноним 02/01/23 Пнд 16:18:02 570209 526
>>570204
дубынина про алкоголь глянь для начала. вкрации - спирт подобен гамку и дофамину вроде
Аноним 02/01/23 Пнд 17:29:58 570213 527
>>570209
Ну а ИТТ ответить просто и понятно разве нельзя?
Аноним 02/01/23 Пнд 17:51:21 570214 528
>>570204
>Алкоголь - это спирт, в определённом смысле, щёлочь.
Это не так. Иначе тогда серная кислота тоже щелочь, ведь в ней есть -OH группы
Аноним 02/01/23 Пнд 18:21:03 570216 529
>>570214
>Это не так. Иначе тогда серная кислота тоже щелочь, ведь в ней есть -OH группы
Н[2]-(О[2]-S=O[2]) это нихуя не тоже самое, что и какой-нибудь K-OH или (H-(C=H[2])-(C=H[2]))-OH
Тем более, это всё равно не ответ на вопрос как спирт в организме приводит к опьянению.
Аноним 02/01/23 Пнд 18:30:33 570220 530
>>567887 (OP)
Кто-то может объяснить на пальцах, почему размерность постоянной Авогадро равна моль^(-1)?
Аноним 02/01/23 Пнд 18:30:41 570221 531
>>570216
SO2(OH)2 вариант записи серной кислоты

>всё равно не ответ на вопрос как спирт в организме приводит к опьянению
вкрации - спирт подобен гамку и дофамину вроде
Тебе уже отвечали
Аноним 02/01/23 Пнд 18:36:48 570222 532
>>570221
>вкрации - спирт подобен гамку и дофамину вроде
>Тебе уже отвечали
Я не химик же, я мимокрок, поэтому и переспрашиваю в треде тупых вопросов с целью получить простой и доступный ответ
Аноним 02/01/23 Пнд 19:42:12 570227 533
>>570222
Мешает просто работать нейронам в голове в обычном темпе. Где-то ускоряет, где-то замедляет.
Аноним 02/01/23 Пнд 21:54:46 570232 534
>>570213
я не могу, потому что не спец и не помню, чё он говорил, да и лучше Дубынина трудно разъяснить это в принципе, наверно. пиздец зумеры пошли, уже видосы короткие смотреть лень
Аноним 03/01/23 Втр 02:32:29 570238 535
>>570186
Како ты долбаеб пздц прст!
Аноним 03/01/23 Втр 07:36:51 570246 536
Аноним 03/01/23 Втр 11:05:34 570254 537
>>570246
Его же уже сто шестьдесят лет как запустили. Там же проблема не в том чтоб запустить, а чтоб в выхлопе было больше энергии чем потрачено.
Аноним 03/01/23 Втр 14:59:02 570269 538
Как алкоголь влияет на мышление и чувства человека? Он создаёт иллюзию достигнутого успеха? Хотелось бы поподробнее узнать об этом
Аноним 03/01/23 Втр 15:49:15 570271 539
>>570269
Он снимает тормоза.
мимонеупотребляющий
Аноним 03/01/23 Втр 18:11:15 570274 540
image.png 1Кб, 147x44
147x44
Откуда берется эта формула? И я правильно понял, что понятие угловой скорости вводится только для кругового движения? А какое-нибудь движение по эллипсу или по кривой линии нужно сводить к движению по окружности с радиусом равным радиусу кривизны очень маленького участка эллипса или кривой траектории, на котором радиус кривизны не меняется?
Аноним 03/01/23 Втр 21:19:34 570282 541
>>570274
Берется из определения угловой скорости и свойства перевода сферических координат в декартовые (конкретно как дифференциалы переводятся), но оно вообще чисто интуитивно понятно, если ты нарисуешь все три вектора.
Аноним 04/01/23 Срд 08:34:49 570292 542
>>570274
Как сказал анон выше, нарисуй картинку. Фундаментальные причины самой формулы кроются в математике (а именно в том, что векторное произведение соответствует бесконечно малым порождающим поворотов, потому что оно индуцирует алгебру Ли, изоморфную SO(3)).
Аноним 04/01/23 Срд 10:50:22 570294 543
Аноним 04/01/23 Срд 15:36:04 570304 544
ввв.jpg 169Кб, 1086x844
1086x844
Сап, по условию тело движется со скоростью v (это вектор), и за время dt получает приращение скорости dv
вот это на пикриле показано.
И там написано что проекция равна приращению модуля скорости. Не совсем понимаю почему, мне кажется новая скорость должна изображаться в виде вектора, достроенного по правилу треугольника, и тогда приращение длины вектора скорости (то же что приращение модуля скорости как я понимаю) будет не совпадать с этим отрезком который фигурной скобкой отмечен.
Аноним 04/01/23 Срд 23:56:33 570318 545
>>570304
Накати курс линейной алгербы.
dv и v - вектора из разных пространств, поэтому когда они вместе, нужны особые операции.
Квадрат модуля это билинейный объект, следовательно его дифференциал тоже должен быть билинейным объектом.
Аноним 05/01/23 Чтв 04:21:49 570326 546
>>570304
>достроенного по правилу треугольника
У тебя одна вещь это скаляр, а вторая это бесконечно малый кусочек. Если ты по правилу треугольника достроишь, то у тебя всё приращение в проекции будет порядка dv, а приращение, связанное с изменением направления будет второго порядка малости, т.е. в пределе это бесконечно малое от бесконечно малого. Можешь сам проверить, если хочешь в обратную сторону, возьми производную от модуля, у тебя вылезет выражение вида v dv, или вообще через предел сам посчитай.
Аноним 05/01/23 Чтв 08:14:32 570332 547
>>570326
А вот в случае равномерного движения по кругу v и dv были бы перпендикулярны, и приращение в проекции было бы равно нулю, что бы значило что скорость изменяется только по направлению. Так? Кажется теперь понятнее.
Аноним 09/01/23 Пнд 16:28:46 570541 548
>>567887 (OP)
Тупач расскажи как ставлять ссылку с ютуб штоб "раскрыть" было?
Аноним 23/03/23 Чтв 14:34:13 574379 549
>>568451
Всё дело в таком явлении как суперстимул (или сверхстимул). Вообще у многих животных встречается, но человек как-то наверное более всех выделился.

Это короче когда модель поведения особи настолько сильно стала определяться стимулами, что эти стимулы сами по себе воздействуют на них независимо от изначального предназначения такого поведения.

Примеров из природы вагон - птицы более активно насиживают более крупное яйцо (поскольку это стимул на взращивание более жизнеспособного потомства) но это на поверку оказывается яйцом гнездовых паразитов (кукушки, например). Мухи-самцы спариваются предпочтительно с самками, источающими больше феромонов, но некоторые паразитические грибы эту тему вкурили и стали паразитировать на самках, чтоб после её смерти источать те самые феромоны, и чтоб самцы потом разносили споры на других самок.

Человек зашёл куда более далеко - фетиш на фигуристые жопки и огромные буфера - из той же самой степи, но ныне считается обыкновенным, хотя если так подумать, то эти суперстимулы особо не благоприятствуют хорошей выживаемости.

Только вот некоторые индивиды уходят очень далеко в такие степи – ноги, первоначально воспринимавшиеся как стимул к поиску здорового партнёра, стали суперстимулом для футфетишистов. Примеров просто тьма – и любовь к нечеловеческим (как по размеру, так и по происхождению) членам, и к трупам, и обнюхиванию трусов, к порно, куча их.
Подытоживая, можно сказать так – у некоторых людей суперстимулы настолько супер, что они начинают действовать не во благо, а иногда и вопреки первоначальному смыслу. Ошибочка в расшифровке протоколов, так сказатб.
Настройки X
Ответить в тред X
15000
Добавить файл/ctrl-v
Стикеры X
Избранное / Топ тредов